FAR Part 3

अब Quizwiz के साथ अपने होमवर्क और परीक्षाओं को एस करें!

Whether recognized or unrecognized in an entity's financial statements, disclosure of the fair values of the entity's financial instruments is required when #It is practicable to estimate those values/ Aggregated fair values are material to the entity A No/No B No/Yes C Yes/No D Yes/Yes

D An entity is required to disclose, either in the body of the financial statements or in the accompanying notes, the fair value of financial instruments for which it is practicable to estimate that value and the method(s) and significant assumptions used to estimate the fair value of financial instruments. These provisions need not be applied to immaterial items.

Gains from remeasuring a foreign subsidiary's financial statements from the local currency, which is not the functional currency, into the parent company's currency should be reported as a(an) A Deferred foreign exchange gain. B Item of other comprehensive income. C Extraordinary item, net of income taxes. D Part of continuing operations.

D If an entity does not maintain its books in its functional currency, remeasuring into the functional currency is required prior to translation into the reporting currency (i.e., the parent company's currency). *Exchange gains and losses that result for the remeasuring process are recognized in income from continuing operations.*

The following information pertains to a sale and leaseback of equipment by Mega Co. on December 31 of the current year: Sales Price$400,000Carrying amount300,000Monthly lease payment3,250Present value of lease payments36,900Estimated remaining life25 years Lease term1 year Implicit rate 12% What amount of gain on the sale should Mega report at December 31 of the current year? A$0 B$36,900 C$63,100 D$100,000

D Mega concludes that the transfer of the tower assets is a sale. Mega's sale price to be $400,000 and CV is $300,000. The Journal entry to record sale is: Dr.Cash400,000 Cr.Asset300,000 Cr.Gain on Sale100.000

When remeasuring foreign currency financial statements into the functional currency, which two of the following items would be remeasured using historical exchange rates? Inventories carried at cost. Equity securities reported at market values. Bonds payable. Accrued liabilities. Prepaid expenses. A I and II B II and V C II and III D I and V

D Nonmonetary balance sheet items, including intangibles, fixed assets, inventory, and prepaid expenses, are remeasured at historical rates.

Mr. and Mrs. Dart own a majority of the outstanding capital stock of Wall Corp., Black Co., and West, Inc. During the year, Wall advanced cash to Black and West in the amount of $50,000 and $80,000, respectively. West advanced $70,000 in cash to Black. At December 31, none of the advances was repaid. In the combined December 31 balance sheet of these companies, what amount would be reported as receivables from affiliates? A $200,000 B $130,000 C $ 60,000 D $0

D) Like consolidated financial statements, combined financial statements should not include intercompany payables and receivables.

Which of the following phrases best describes a Level 1 input for measuring the fair value of an asset or liability? A Inputs for the asset or liability based on the reporting entity's internal data. B Quoted prices for similar assets or liabilities in active markets. C Inputs that are principally derived from or corroborated by observable market data. D Unadjusted quoted prices for identical assets or liabilities in active markets.

The correct answer is (D). There are 3 levels of input for measuring the fair value of an asset or liability. Level 1- Unadjusted quoted prices for identical assets or liabilities in active markets. Level 2: Inputs other than quoted prices included within Level 1 that are observable for similar assets or liabilities, either directly or indirectly. Level 3: Unobservable inputs for the asset or liability. (A), (B) and (C) are incorrect because they are Level 2 inputs.

Zest Co. owns 100% of Cinn, Inc. On January 2, Zest sold equipment with an original cost of $80,000 and a carrying amount of $48,000 to Cinn for $72,000. Zest had been depreciating the equipment over a five-year period using straight-line depreciation with no residual value. Cinn is using straight-line depreciation over three years with no residual value. In Zest's December 31, consolidating worksheet, by what amount should depreciation expense be decreased? A $ 0 B $ 8,000 C $16,000 D $24,000

The equipment was being depreciated by the seller at a rate of $16,000 per year ($80,000 original cost / 5 years). The buyer will record depreciation expense of $24,000 ($72,000 cost / 3 years). = 24,000 -16,000 = 8,000 With intercompany sales of fixed assets, in the year of the sale the carrying amount of the asset is restored to its original book value and the gain (loss) recorded by the seller is eliminated. no gain or loss has taken place for the consolidated entity; assets merely have been transferred from one set of books to another.

A foreign subsidiary's functional currency is its local currency, which has not experienced significant inflation. The weighted average exchange rate for the current year would be the appropriate exchange rate for translating #Salaries expense/Sales to external customers A Yes/Yes B Yes/No C No/Yes D No/No

A Foreign currency financial statements should be translated by means of the following rates: **all assets and liabilities at the current exchange rate at the balance sheet date**; ++revenues and expenses at the exchange rate at the time the revenue or expense was recognized, however, due to the impracticability of this where rates change frequently. A weighted-average exchange rate for the period may be used--; +contributed capital+ at the +historical exchange rate+; and @retained earnings at the translated amount of retained earnings for the prior period@ +/- net income(loss) at the weighted-average rate - dividends declared during the period at the exchange rate when declared. The weighted average exchange rate would be an appropriate exchange rate for salaries expense and sales to external customers.

Troop Co. frequently borrows from the bank to maintain sufficient operating cash. The following loans were at a 12% interest rate, with interest payable at maturity. Troop repaid each loan on its scheduled maturity date. Date of LoanAmount Maturity date Term of loan 11/1, Yr 5 10,000 10/31, Yr 6 1 year 2/1, Yr 6 30,000 7/31, Yr 6 6 months 5/1, Yr 6 16,000 1/31, Yr 7 9 months Troop records interest expense when the loans are repaid, thus interest expense of $3,000 was recorded in year 6. If no correction is made, by what amount would year 6 interest expense be understated? A $1,080 B $1,240 C $1,280 D $1,440

A Interest expense should be accrued in the period in which it is earned, rather than the period in which it is paid. In year 6, the total interest expense of $4,080 (1,000+1,800+1,280) should be reported, as shown below. If only $3,000 interest expense is recorded, the understatement is $1,080(4,080-3,000).

How should a company report its decision to change from a cash-basis of accounting to accrual-basis of accounting? A As a change in accounting principle, requiring the cumulative effect of the change (net of tax) to be reported in the income statement. B Prospectively, with no amounts restated and no cumulative adjustment. C As an extraordinary item (net of tax). D As a prior-period adjustment (net of tax), by adjusting the beginning balance of retained earnings.

A The change from cash-basis accounting to accrual-basis accounting is a change from an accounting principle that is not generally accepted to one that is generally accepted, which is considered a correction of an error. Correction of an error requires a prior-period adjustment, which is done net of tax, by adjusting the beginning retained earnings.

Lore Co. changed from the cash basis of accounting to the accrual basis of accounting during the year. The cumulative effect of this change should be reported in Lore's financial statements as a A Correction of an error. B Change in accounting principle. C Component of income before extraordinary item. D Component of income after extraordinary item.

A The change from the cash basis of accounting (not GAAP) to the accrual basis of accounting (GAAP) is a correction of an error. The correction of an error in prior period income is reported as a prior-period adjustment by restating prior-period financial statements.

On April 1, Dart Co. paid $620,000 for all the issued and outstanding common stock of Wall Corp. in a transaction properly accounted for as a business combination. The recorded assets and liabilities of Wall Corp. on April 1 follow: Cash $ 60,000 Inventory 180,000 Property and equipment (net of accumulated depreciation of $220,000) 320,000 Liabilities (120,000) Net assets $440,000 On April 1, Wall's inventory had a fair value of $150,000, and the property and equipment (net) had a fair value of $380,000. What is the amount of goodwill resulting from the business combination? A $150,000 B $120,000 C $ 50,000 D $ 20,000

A) 150,000(620,000-(440,000-180,000+150,000-320,000+380,000)) Goodwill is the excess of the investment cost, plus the fair value of any noncontrolling interest, over the fair value of the identifiable net assets acquired.

Wagner, a holder of a $1,000,000 Palmer, Inc. bond, collected the interest due on March 31 of the current year, and then sold the bond to Seal, Inc. for $975,000. On that date, Palmer, a 75% owner of Seal, had a $1,075,000 carrying amount for this bond. What was the effect of Seal's purchase of Palmer's bond on the retained earnings and noncontrolling interest amounts reported in Palmer's March 31 consolidated balance sheet? #Retained earnings/Noncontrolling interest A $100,000 increase/$ 0 B $ 75,000 increase/$ 25,000 increase C $ 0/$ 25,000 increase D $ 0/$100,000 increase

A) An investment by one member of a consolidated group of companies in the bonds of another member of that group is, in substance, the same thing as the purchase by a member of its own bonds. Treat like retirement of bond. Recognizes a gain $100,000 (1,075,000-975000) on the bond retirement on the book of the issuer. There are no gain is attributed to subsidiary = unaffected.

Which of the following statements regarding foreign exchange gains and losses is correct? A An exchange gain occurs when the exchange rate increases between the date a payable is recorded and the date of cash payment. B An exchange gain occurs when the exchange rate increases between the date a receivable is recorded and the date of cash receipt. C An exchange loss occurs when the exchange rate decreases between the date a payable is recorded and the date of the cash payment. D An exchange loss occurs when the exchange rate increases between the date a receivable is recorded and the date of the cash receipt.

B Receivable: Rate increase results in a gain (receive more at settlement), rate decrease results in a loss (receive less at settlement). Payable: Rate increase results in a loss (pay more at settlement), rate decrease results in a gain (pay less at settlement).

A foreign subsidiary of a U.S. parent company should measure its assets, liabilities and operations using A The subsidiary's local currency B The subsidiary's functional currency C The U.S. dollar D The best available spot rate

B The assets, liabilities, and operations of a foreign subsidiary of a U.S. parent company should be *measured in its functional currency*. An entity's functional currency is the currency of the primary economic environment in which the entity operates; normally, that is the currency of the environment in which an entity primarily generates and expends cash. The functional currency of a foreign subsidiary may be its local currency, the U.S. dollar, or another foreign currency.

A material overstatement in ending inventory was discovered after the year-end financial statements of a company were issued to the public. What effect did this error have on the year-end financial statements? #Current assets/Gross profit A Understated/Overstated B Overstated/Overstated C Understated/Understated D Overstated/Understated

B The current assets portion of the balance sheet would be overstated because the inventory account is part of current assets. Gross profit would be overstated because an overstatement in inventory results in an understatement of cost of goods sold. The ending inventory is subtracted from goods available for sale to get cost of goods sold. If costs of goods sold is understated, then gross profit is overstated.

A balance arising from the translation or remeasurement of a subsidiary's foreign currency financial statements is reported in the consolidated income statement when the subsidiary's functional currency is the: #Foreign currency/U.S. dollar A No/No B No/Yes C Yes/No D Yes/Yes

B The functional currency of a foreign entity may be its local currency (i.e., the foreign currency) or the reporting currency (e.g., the U.S. dollar). ++If the functional currency is the foreign currency, the foreign currency financial statements must be translated into U.S. dollars.++ The translation adjustments which result from this process are not reported in the consolidated income statement but are reported in +other comprehensive income.+ @@If the functional currency is the U.S. dollar, the foreign currency financial statements must be remeasured into U.S. dollars.@@ The foreign exchange gains/losses which result from this process are reported in the @@consolidated income statement.@@

Which of the following expenses related to the business combination should be included, in total, in the determination of net income of the combined corporation for the period in which the expenses are incurred? #Fee of finders and consultants #Registration fees for equity securities issued A Yes/Yes B Yes/No C No/Yes D No/No

B)Those costs shall include finders' fees; advisory, legal, accounting, valuation, and other professional or consulting fees. General administrative costs, including the costs of maintaining an internal acquisitions department; and costs of registering and issuing debt and equity securities. The acquirer shall account for acquisition-related costs as expenses. Costs of registering and issuing any equity securities are treated as a reduction fair value of the equity securities.

On January 1, Dallas, Inc. purchased 80% of Style, Inc.'s outstanding common stock for $120,000. On that date, the carrying amounts of Style's assets and liabilities approximated their fair values. During the year, Style paid $5,000 cash dividends to its stockholders. Summarized balance sheet information for the two companies follows: Dallas Style 12/31 12/31 1/01 Investment in Style (equity method)$132,000 Other assets 138,000 $115,000 $100,000 $270,000 $115,000 $100,000 Common stock $ 50,000 $20,000 $20,000 Additional paid-in capital 80,250 44,000 44,000 Retained earnings 139,750 51,000 36,000 $270,000 $115,000 $100,000 What amount should Dallas report as earnings from subsidiary in its year-end income statement? A $12,000 B $15,000 C $16,000 D $20,000

Beg Retained Earnings + Net Income - Dividends = Ending Retained Earnings Net Income = Ending Retained Earnings + Dividends - Beg Retained EarningsNet Income = 51,000 + 5,000 - 36,000 Net Income = 20,000 0.8*20,000 = 16,000

Which of the following items would not be considered a change in accounting estimate? A A change in the useful lives or salvage values of depreciable assets B A change in the recovery periods benefited by a deferred cost C A change in the method of accounting for long-term construction contracts D A change in expected losses on receivables

C A change in the method of accounting for long-term construction contracts would be considered a change in accounting principle, not a change in accounting estimate.

Which of the following is not a characteristic of market participants when measuring fair value? A They are independent of each other. B They are knowledgeable and sufficiently informed about the transaction. C They are related parties. D They are willing and able to enter into the transaction. Market participants are buyers and sellers in the market for the asset or liability that are independent of the reporting entity, knowledgeable, willing and able to transact for the asset or liability (not forced).

C Market participants are buyers and sellers in the market for the asset or liability that are *independent of the reporting entity*, *knowledgeable, willing and able to transact for the asset or liability (not forced)*.

Thyme, Inc. owns 16,000 of Sage Co.'s 20,000 outstanding common shares. The carrying value of Sage Co's equity is $500,000. Sage subsequently issues an additional 5,000 previously unissued shares for $200,000 to an outside party that is unrelated to either Thyme or Sage. What is the total non-controlling interest after the additional shares are issued? A $140,000 B $172,000 C $252,000 D $300,000

C) Non-controlling Interest = 9,000 (20,000-16,000+5,000) / 25,000 (20000+5000) = 36% Total equity of Sage = $700,000 (500,000 + 200,000). Value of Non-Controlling Interest = Non-Controlling Interest x Total equity of Sage = $700,000 x 36% = $252,000. Note: **Non-controlling interest is the portion of the subsidiary's equity held by minority shareholders who do not have control over the company.**

Which of the following statements is correct regarding accounting changes that result in financial statements that are, in effect, the statements of a different reporting entity? A Cumulative-effect adjustments should be reported as separate items on the financial statements pertaining to the year of change. B No restatements or adjustments are required if the changes involve consolidated methods of accounting for subsidiaries. C No restatements or adjustments are required if the changes involve the cost or equity methods of accounting for investments. D The financial statements of all prior periods presented should be reported by retrospective application. Retrospective application based on the earliest period presented is provided for an accounting change that is change in reporting entity. Here, the term "restatement" refers only to correction of errors in previously issued financial statements.

D Retrospective application based on the earliest period presented is provided for an accounting change that is change in reporting entity. Here, the term "restatement" refers only to correction of errors in previously issued financial statements.

In a transaction accounted for as a business combination, the appraised values of the identifiable net assets acquired exceeded the acquisition price. How should the excess appraised value be reported? A As negative goodwill B In Income from Discontinued Operations C In Other Comprehensive Income D In Income from Continuing Operations

D) In a business combination, if the appraised values of the identifiable assets acquired exceeded the acquisition price, it would lead to a credit balance (a negative goodwill) which is recorded as a gain on the income statement from Continuing Operations.

When a company changes the expected service life of an asset because additional information has been obtained, which of the following should be reported? #Pro forma effects of retroactive application/ Cumulative effect of a change in accounting principle A Yes/Yes B No/Yes C Yes/No D No/No

D A change in the expected service life of an asset because additional information has been obtained is an example of a change in an accounting estimate. A change in estimate should be accounted for prospectively. Therefore, no cumulative effect of the change is reported and no pro forma effects of retroactive application are disclosed.

What if the asset has a cash price 95,000 FV 100,000

Dr. Lease Property 95000 Cr. Lease Liability 95000 Can't record an asset higher than it's cash price.

A shareholder holds 100 shares of a company and anticipates a price fall in the future. To protect itself from this future event, which of the following actions can he take? A Sell a call option B Buy a call option C Sell a put option D Buy a put option

D A holder of a put option has the right to sell an underlying asset in the future. It gives the shareholder the right to sell the shares at a pre-determined price (strike price) at a given date. The shareholder will benefit if the price of the shares falls as predicted since he has an option to sell the shares at the strike price. Note that this option should be exercised only if the actual share price falls below the strike price. If not, the holder will let the option expire since the holder would prefer to sell it at the actual price, which is more than the strike price.

Sayon Co. issues 200,000 shares of $5 par value common stock to acquire Trask Co. in an acquisition method business combination. The market value of Sayon's common stock is $12. Legal and consulting fees incurred in relationship to the purchase are $110,000. Registration and issuance costs for the common stock are $35,000. What should be recorded in Sayon's additional paid-in capital account for this business combination? A $1,545,000 B $1,400,000 C $1,255,000 D $1,365,000

D Dr Investment in Trask $2,400,000 Cr Cash 35,000 Cr Common Stock 1,000,000 Cr Additional paid-in-capital (APIC) 1,365,000

Which of the following statements is *false concerning subsequent events? A An entity shall not recognize events occurring between the time the financial statements were issued or were available to be issued and the time the financial statements were reissued unless the adjustment is required by GAAP or regulatory requirements. B Some nonrecognized subsequent events may be of such a nature that they must be disclosed to keep the financial statements from being misleading. C An entity shall not recognize subsequent events that provide evidence about conditions that did not exist at the date of the balance sheet but arose after the balance sheet date but before financial statements are issued or are available to be issued. D An entity is not permitted to disclose any estimates inherent in the process of preparing financial statements when disclosing the effects of subsequent events that provide additional evidence about conditions existing at the date of the balance sheet.

D) An entity shall recognize in the financial statements the effects of all subsequent events that provide additional evidence about conditions that existed at the date of the balance sheet, including the estimates inherent in the process of preparing financial statements.

The disclosure requirement associated with the date through which management has evaluated subsequent events, along with the basis for that date being the appropriate date, needs to be implemented in financial statements without regard to which of the following? A Whether the statements have been compiled. B Whether the statements have been prepared using an Other Comprehensive Basis of Accounting. C Whether the statements are full disclosure. D Both A. and B.

D) Full-disclosure financial statements without the statements have been audited, reviewed, or compiled. Essentially, the disclosure is required regardless of the level of service performed on the financial statements, and regardless of the basis of accounting used in preparing the statements.

On February 1, Pizza, Inc. acquired 75% of the outstanding common stock of Sausage Co. for $750,000 cash. At February 1, Sausage's balance sheet showed a carrying amount of net assets of $1,100,000 and the fair value of Sausage's assets and liabilities equaled their carrying amounts except for property, plant, and equipment which exceeded its carrying amount by $200,000. On February 1, what amount would be attributed as fair value for the noncontrolling interest? A $325,000 B $300,000 C $275,000 D $250,000

D) Pizza paid $750,000 for 75% of Sausage, thus 25% is considered as noncontrolling interest. $750,000 ÷ .75 = $1,000,000; then $1,000,000 × .25 = $250,000 fair value for the noncontrolling interest.

An entity disposes of a nonmonetary asset in a nonreciprocal transfer. A gain or loss should be recognized on the disposition of the asset when the fair value of the asset transferred is determinable and the nonreciprocal transfer is to #Another entity/A stockholder of the entity A No/Yes B No/No C Yes/No D Yes/Yes

D)A transfer of a nonmonetary asset to a stockholder or to another entity in a nonreciprocal transfer should be recorded at the fair value of the asset transferred, and a gain or loss should be recognized on the disposition of the asset.

On December 31, year 1, Roe Co. leased a machine from Colt for a five-year period. Equal annual payments under the lease are $100,000 and are due on December 31 of each year. The first payment was made on December 31, year 1, and the second payment was made on December 31, year 2. The five lease payments are discounted at 10% over the lease term. The present value of lease payments at the inception of the lease and before the first annual payment was $417,000. The lease is appropriately accounted for as a Finance Lease by Roe. In its December 31, year 2 balance sheet, Roe should report a lease liability of A$317,000 B$315,000 C$285,300 D$248,700

Roe's annual lease payment is$100,000. Balance before payment, 12/31, year 1 $417,000 Less MLP, 12/31, year 1 (100,000) Balance after MLP, 12/31, year 1 $317,000 Minimum lease payment, 12/31, year 2$100,000 Interest expense ($317,000 × 10%) ( 31,700) Less: Principal Reduction (68,300) Balance after MLP, 12/31, year 2 $248,700

Bensol Co. and Sable Co. exchanged similar trucks with fair values in excess of carrying amounts. In addition, Bensol paid Sable to compensate for the difference in truck values. As a consequence of the exchange, which lacked commercial substance, Sable recognizes A A gain equal to the difference between the fair value and carrying amount of the truck given up. B A gain determined by the proportion of cash received to the total consideration. C A loss determined by the proportion of cash received to the total consideration. D Neither a gain nor a loss.

Sable exchanged a truck with a fair value in excess of its carrying amount for a similar truck and received monetary consideration. Hence, Sable experiences a gain on the exchange. The recipient of monetary consideration in an exchange measured based on the recorded amount recognizes recognizes a portion of the gain experienced. The amount of the gain recognized is based on the ratio of the monetary consideration to the total consideration received.

On December 31, Year 1, Bit Co. had capitalized costs for a new computer software product with an economic life of five years. Sales for year 2 were 30 percent of expected total sales of the software. At December 31, Year 2, the software had a net realizable value equal to 90 percent of the capitalized cost. What percentage of the original capitalized cost should be reported as the net amount on Bit's December 31, Year 2, balance sheet? A 70% B 72% C 80% D 90%

The annual amortization of the capitalized software cost is the greater of: (1) the ratio of current revenues to current and future revenues (e.g., 30%) or (2) the straight-line method over the remaining useful life of the software including the period to be reported upon (e.g., 1 / 5 = 20%). Because the software has a net realizable value of 90% of the capitalized cost, it can be reported on the balance sheet at 70% (i.e., 1 - 30%) of its capitalized cost.

Which of the following computer software costs should be expensed? A Conceptual formulation of alternatives in the preliminary project stage B Design of the software configuration during the application development stage C Costs of producing product masters after technology feasibility was established D Installation to hardware during the application development stage

The correct answer is (A). In the case of computer software developed for sale, costs prior to technological feasibility are expensed as R&D and costs associated with converting a technologically feasible program into final commercial form are capitalized. Costs incurred after software sales begin are inventories and included in COGS. Computer software costs that are incurred in the preliminary project stage should be expensed as incurred. Activities in this stage include conceptual formulation and evaluation of alternatives; determination of the existence of needed technology; and final selection of alternatives. The following should be capitalized: Design of the software configuration during the application development stage Costs of producing product masters after technology feasibility was established Installation to hardware during the application development stage

A collection agency spent $50,000 in staff payroll costs investigating the feasibility of developing its own software program for tracking customer contacts. After committing to funding the project, software developers were paid $200,000 to write the code, and the company incurred $70,000 in general and administrative costs related to training and software maintenance. What amount should be capitalized? A $200,000 B $250,000 C $270,000 D $320,000

The correct answer is (A). Software development process: 1. Preliminary project stage: All costs in this stage, including investigating the feasibility of developing its own software, should be expensed. Hence, $50,000in staff payroll costs investigating the feasibility of developing its own software program should be expensed here. 2. Application development stage: **Costs incurred in the application development stage, including writing the code, are usually capitalized**, except for training costs, which are expensed. The $200,000 paid to write the code should be capitalized whereas $70,000 in general and administrative costs related to training and software maintenance should be expensed. 3. Post-implementation stage: Once the product is market ready and in saleable condition, all costs should be expensed.

The primary criteria for determining a fair value hedge includes the fact that the hedged item does which of the following? A The hedged item is specifically identified as either all or a specific portion of a recognized asset or liability or of an unrecognized firm commitment. B All answer choices are correct. C The hedged item is a single asset or liability (or a specific portion thereof) or is a portfolio of similar assets or a portfolio of similar liabilities. D The hedged item presents an exposure to changes in fair value attributable to the hedged risk that could affect reported earnings

The correct answer is (B). An asset or a liability is eligible for designation as a hedged item in a fair value hedge if all of the following additional criteria are met: The hedged item is specifically identified as either all or a specific portion of a recognized asset or liability or of an unrecognized firm commitment. The hedged item is a single asset or liability (or a specific portion thereof) or is a portfolio of similar assets or a portfolio of similar liabilities. The hedged item presents an exposure to changes in fair value attributable to the hedged risk that could affect reported earnings. For determining a fair value hedge includes the fact that the hedged item does all of the above.

On December 31, year 3, Byte Co. had capitalized software costs of $600,000 with an economic life of four years. Sales for year 4 were 10% of the expected total sales of the software. At December 31, year 4, the software had a net realizable value of $480,000. In its December 31, year 4 balance sheet, what amount should Byte report as net capitalized cost of computer software? A $432,000 B $450,000 C $480,000 D $540,000

The correct answer is (B). The net capitalized cost of computer software = $450,000. In the case of computer software that is developed for sale, lease or to market as a product, the costs incurred to convert a technologically-feasible program into a final commercial product are capitalized. The capitalized costs are then amortized annually to arrive at the carrying value which is then compared with the net realizable value for reporting purposes. Annual amortization = Greater of SLM (Capitalized Amount / Estimate of economic life) Units of sales [Capitalized Amount x (Revenues during the year / Total revenues expected)]. The software capitalized cost of Byte Co. on December 31, year 3 = $600,000. Annual amortized cost for year 4 = $600,000 / 4 = $150,000. (Since SLM > Units of Sales) Carrying value on December 31, year 4 = $600,000 - $150,000 = $450,000. Net realizable value on December 31, year 4 = $480,000.

Which of the following common characteristics of derivative financial instruments distinguishes them from other types of financial instruments? A They impose a contractual obligation by one entity to deliver cash to a second entity to convey a contractual right. B They are financial investments in stocks, bonds, or other securities that are marketable. C They have a notional amount or payment provision that is based on the changes in one or more underlying variables. D Most financial instruments are valued on the balance sheet at fair value, but derivatives are valued on the balance sheet at cost.

The correct answer is (C) A derivative is a financial instrument or other contract that derives its value from the movement of prices, interest rates, or foreign exchange rates associated with an underlying asset or financial instrument. A notional amount is a number of currency units, shares, bushels, pounds, or other units specified in the contract. The notional amount is called a face amount in some contracts. A payment provision specifies a fixed or determinable settlement to be made if the underlying behaves in a specified manner. Derivatives are the only financial instruments to have such notional amounts or payment provisions based on underlying variables. Contractual rights and obligations are not unique to derivatives and are features of other financial instruments as well. Derivatives are valued on the balance sheet at their fair values and not at cost.

Which of the following statements about derivatives is incorrect? A Derivatives are settled in net B Derivatives have an underlying asset which determines the value of the instrument C All derivatives are required to be listed D They have a negligible initial investment

The correct answer is (C). Derivatives are characterized by the following main features: The requirement of a very small or negligible investment fee Option to settle the transaction at a net amount An underlying asset that decides the settlement amount of the derivative However, there is no requirement for the derivatives to be listed under US GAAP. Derivatives may very well be traded without being listed.

Under IFRS (IAS17, Leases), which of the following is not a criterion for a finance lease? A The present value of the lease payments is equal to substantially all of the fair value of the asset B There is a bargain purchase option C Cancellation losses are borne by the lessor D The asset is of a specialized nature, useful only to the lessee without modification

The correct answer is (C). Cancellation losses are borne by the lessee, not the lessor, for financing leases under IFRS. The remaining answer choices are all criteria for a finance lease under IFRS.

On January 1, year 1, a company purchased for $10,000 an at-the-money call option on 1,200 barrels of crude oil, which the company intends to purchase in five years. The company elected to exclude the time value of the option from the assessment of effectiveness, classified the option as a cash flow hedge, and applied a straight-line amortization to the initial option premium. On December 31, year 1, the time value of the option decreased by $1,200, and the change in intrinsic value increased by $1,800. The journal entry that the company should make on December 31, year 1, to record the change in the value of the derivative should include which of the following as a credit? A Derivative asset, $600. B Derivative asset, $1,400. C Other comprehensive income, $600. D Other comprehensive income, $1,400.

The correct answer is (C). The effective and ineffective portion of gains and losses on cash flow derivative instruments is reported as a component of OCI and reclassified into earnings when the hedged forecasted transaction affects earnings. Change in Value of Derivative = Decrease in Time Value ($1,200) + Increase in Intrinsic Value ($1,800) Change in Value of Derivative = $600, Increase. This increase in the value of the derivative of $600, which is designated as a cash flow hedge, would be reported with a credit in Other Comprehensive Income with the following journal entry: Dr. Derivative asset$600 Cr. Other comprehensive income$600

Grey Co. purchased stock in Cherry Co. Grey purchased a put option on the stock. The strike price is the current market price. What is the most likely reason Grey purchased the put option? A Cherry stock has remained flat, and Grey believes the stock is going to remain at its original purchase price. B Cherry stock has increased in price, and Grey believes the stock is going to continue to increase in price. C Cherry stock has decreased in price, but Grey believes the stock is going to increase in price. D Cherry stock has increased in price, but Grey is concerned that the price might decrease.

The correct answer is (D) Buying a put option confers the right on the holder of the option to sell the stock at a price, regardless of the trading price of the underlying asset/stock. In essence, the put option buyer buys the right to sell the underlying stock to the put option holder at a predetermined rate. Hence, put options are purchased mainly when the buyer is bearish and aims at hedging itself of price falls in the future.Since Grey Co. has purchased a put option on the Cherry stocks, it means that the company is concerned that the price might decrease.

n January 1, year 1, a company capitalized $100,000 of costs for software that is to be sold. The company amortizes the software costs on a straight-line basis over five years. The carrying value of the software costs on January 1, year 3, was $60,000. As of December 31, year 3, the estimated future gross revenue to be generated from the sale of the software is $23,000, and the estimated future cost of disposing of the software is $8,000. What amount should the company expense related to the software costs for the year ended December 31, year 3? A $18,400 B $20,000 C $37,000 D $45,000

The correct answer is (D). Software production costs are capitalized and reported at the lower of unamortized cost or net realizable value (NRV) at the point of technological feasibility for the software. Net realizable value is the value of an asset that can be realized upon the sale of the asset, less a reasonable estimate of the costs associated with either the eventual sale or the disposal of the asset in question. Depreciation on software = Capitalized cost of software/Useful life = $100,000/5 = $20,000. Total expenses related to software costs are depreciation charges and excess of the CV over the NRV reported at $45,000 ($20,000 + $25,000). Cost of the software purchased 2 years ago $100,000 Carrying value of the software at the beginning of the year ($100,000 - $20,000 - $20,000) $60,000 Depreciation for the year ($20,000 ) $20,000 Carrying value at the end of the year ($60,000 - $20,000) $40,000 Expected gross revenue from sale of the software $23,000 Expected future costs to dispose the software $8,000 Net realizable value (NRV) ($23,000 - $8,000) $15,000 Excess of carry value over the NRV ($40,000 - $15,000) $25,000 Total expense for the year ($20,000 Depreciation + $25,000 Impairment). $45,000

Which of the following is not a criterion for the lessor to classify a lease as either a sales-type or direct-financing-type lease? A The lease is a finance lease for the lessee. B The lease transfers ownership of the underlying asset to the lessee by the end of the lease term. C The lease grants the lessee an option to purchase the underlying asset that the lessee is reasonably certain to exercise. D The underlying asset is of such a specialized nature that it is expected to have no alternative use to the lessor at the end of the lease term.

"The lease is a finance lease for the lessee" is not a criterion for determination of lease classification by the lessor. A lessor should classify a lease as a sales-type lease when the lease meets any one of the following criteria at lease commencement (these are the same criteria for a Finance Lease): -Present Value equals or exceeds substantially all (90%) of the Fair Value -Option to Purchase (exercise is reasonably certain) -Economic Life - Major part (75%) of asset's economic life is used -Transfer of Ownership at lease termination -Specialized Nature - No alternative use to the lessor at lease termination

The net investment in the lease is measured as the present value of the sum of the:

(1) Lease payments not yet received by the lessor, (2) +Difference Guaranteed Residual value and Expected Residual Value (3) +Bargain Purchase Option (4) - Selling Profit

Organization costs include

(1) Organization *accounting services* (2) *legal services* for drafting the corporate charter and bylaws, (3) state incorporation *filing fees* (4) *costs of temporary directors and of organizational meetings*. Start-up activities, including organization costs, should be expensed as incurred. **Organization costs are written off over 60 months for tax purposes**

West, Inc. acquired 60% of East Co.'s outstanding common stock. West paid $800,000 to acquire the stock. West plans to relocate East's company headquarters, which is expected to cost between $100,000 and $300,000. The present value of the probability-adjusted relocation cost is $240,000. What is West's acquisition cost? A $800,000 B $900,000 C $1,040,000 D $1,100,000

A

Fogg Co., a U.S. company, contracted to purchase foreign goods. Payment in foreign currency was due one month after the goods were received at Fogg's warehouse. Between the receipt of goods and the time of payment, the exchange rates changed in Fogg's favor. The resulting gain should be included in Fogg's financial statements as a(an) A Component of income from continuing operations. B Extraordinary item. C Deferred credit. D Item of other comprehensive income.

A A change in exchange rates between the functional currency and the currency in which the transaction is denominated increases or decreases the expected amount of functional currency cash flows upon a settlement of the transaction. That increase or decrease in expected functional currency cash flows is a foreign currency transaction gain or loss that generally should be included as a component of income from continuing operations for the period in which the transaction is settled.

Which of the following is the characteristic of a perfect hedge? A No possibility of future gain or loss B No possibility of future gain only C No possibility of future loss only D The possibility of future gain and no future loss

A Hedging is a risk management strategy to protect against the possibility of loss, such as from price fluctuations. Generally, the strategy involves counterbalancing transactions in which a loss on one financial instrument or cash flow stream would be offset by a gain on the related derivative. A perfect hedge would result in no possibility of future gain or loss.

At December 31, year 1, Eagle Corp. reported $1,750,000 of appropriated retained earnings for the construction of a new office building, which was completed in year 2 at a total cost of $1,500,000. In year 2, Eagle appropriated $1,200,000 of retained earnings for the construction of a new plant. Also, $2,000,000 of cash was restricted for the retirement of bonds due in year 3. In its year 2 balance sheet, Eagle should report what amount of appropriated retained earnings? A $1,200,000 B $1,450,000 C $2,950,000 D $3,200,000

A Since the new office building was completed in year 2, retained earnings reported appropriated for that purpose at 12/31/, year 1 should have been returned to unappropriated retained earnings in year 2. Restricted cash does not affect the amount of appropriated retained earnings.

Vik Auto and King Clothier exchanged goods, held for resale, with equal fair values. Each will use the other's goods to promote their own products. The goods exchanged are goods they each would have used otherwise for promotions and thus aren't expected to significantly change future cash flows. The retail price of the car that Vik gave up is less than the retail price of the clothes received. What profit should Vik recognize for the nonmonetary exchange? A A profit is not recognized. B A profit equal to the difference between the retail prices of the clothes received and the car. C A profit equal to the difference between the retail price and the cost of the car. D A profit equal to the difference between the fair value and the cost of the car.

A This transaction does not have commercial substance because it does not significantly change future cash flows. It is an exchange of goods held for resale for other assets to be used not for resale, but merely as a different means of promoting the company's own goods to customers. A nonmonetary exchange without commercial substance is measured based on the recorded amount (after reduction, if appropriate, for an indicated impairment of value) of the nonmonetary asset(s) relinquished, and not on the fair values of the assets exchanged. No gains, only losses, are recognized on exchanges without commercial substance.

A lessee would include which of the following in recording a Finance Lease Asset? A Lease Liability + Initial Direct Cost B Lease Incentives C Prepaid Lease Payments D All of the above

A lessee must record a Finance Lease asset in an amount equal to: Lease liability (Present value of lease payments not yet paid) + Initial direct costs (Costs that are directly attributable to negotiating and arranging the lease that would not have been incurred had the lease not been executed) + Prepaid lease payments - Lease incentives (Include both payments made by the lessor to or on behalf of the lessee and any losses incurred by the lessor as a result of assuming a lessee's preexisting lease with a third party)

Under what circumstances are disclosures required for subsequent events? A Only when the financial statements include notes B Only when financial statements are compiled C Only when financial statements are reviewed D Always

A) The disclosure requirements associated with subsequent events, along with other issues, only need to be included in notes to financial statements when the financial statements include notes.

____________________ in a variable interest entity is a (are) contractual, ownership, or other pecuniary interest(s) in an entity that change(s) with changes in the entity's net asset value exclusive of variable interests. A The primary beneficiary B Variable interests C Subordinated financial support D Expected variability

A) Variable interests in a variable interest entity are contractual, ownership, or other pecuniary interests in an entity that change with changes in the entity's net asset value exclusive of variable interests. A primary beneficiary refers to an enterprise that consolidates a variable interest entity under the provisions of GAAP. Subordinated financial support refers to variable interests that will absorb some or all of an entity's expected losses if they occur. Expected variability is the sum of the absolute values of the expected residual return and the expected loss.

The authoritative guidance on business combinations states which of the following? A A business combination occurs when an acquirer obtains control over one or more businesses. B If intercorporate stock ownership arrangements result in a parent company owning less than 50 percent of a third lower-level subsidiary, through the parent's second subsidiary (a chain of interests), consolidated financial statements should not be prepared. C Consolidation of majority-owned subsidiaries is not required if a large minority interest exists in the subsidiaries. D Consolidation of majority-owned subsidiaries is not required if a subsidiary is in a foreign location.

A)A business combination occurs when an acquirer obtains control over one or more businesses.

GAAP defines ___________________ as any legal structure used to conduct activities or to hold assets. A An entity B A variable interest entity C A business D A special purpose entity

An "entity" is any legal structure used to conduct activities or to hold assets. A "variable interest entity" is defined based on the characteristics of the entity. To be considered a business, the entity must meet the requirements set forth in GAAP. A special purpose entity (SPE) is not recognized in US GAAP.

Appointment of the IFRS Foundation Trustees is approved by which of the following? A The IFRS Advisory Council B The IFRS Interpretations Committee C The IFRS Foundation Monitoring Board D The Accounting Standards Advisory Forum

Appointment of the IFRS Trustees is approved by the IFRS Foundation Monitoring Board. The IFRS Advisory Council is the formal advisory body to the IASB and the Trustees. The IFRS Interpretations Committee reviews current IFRS accounting issues that have arisen and provides authoritative guidance (IFRICs) on those issues. It has 14 voting members appointed by the Trustees of the IFRS Foundation for their technical ability. The Accounting Standards Advisory Forum is an advisory group to the IASB providing technical advice and feedback. Its members consist of national accounting standard-setters and regional bodies with an interest in financial reporting and are selected by the Trustees.

Smythe Co. invested $200 in a call option for 100 shares of Gin Co. $.50 par common stock, when the market price was $10 per share. The option expired in three months and had an exercise price of $9 per share. What was the intrinsic value of the call option at the time of initial investment? A $50 B $100 C $200 D $900

B A call option represents the right, but not the obligation, to buy a set number of shares of stock at a predetermined 'strike price' before the option reaches its expiration date. A call option is purchased in hopes that the underlying stock price will rise in the future well above the strike price. Entities can use the BlackScholes, binomial, or similar pricing models to value the options. In this case, limited information is provided, so the value of the options would be the difference between the current/ market price and the strike/exercise price: 100 * ($10 - $9) = $100.

When there is a change in the reporting entity, how should the change be reported in the financial statements? A Prospectively, including note disclosures B Retrospectively, including note disclosures, and application to all prior period financial statements presented C Currently, including note disclosures D Note disclosures only

B A change in reporting entity occurs when an accounting change results in financial statements that are, in effect, the statements of a different reporting entity. The change is reported by retrospectively applying the change to the financial statements of all prior periods presented to include the financial information for the new reporting entity for the periods. A description of both the nature of the change and the reason for it shall be disclosed in the note disclosures for the period of the change.

A derivative financial instrument is best described as A Evidence of an ownership interest in an entity such as shares of common stock. B A contract that has its settlement value tied to an underlying notional amount. C A contract that conveys to a second entity a right to receive cash from a first entity. D A contract that conveys to a second entity a right to future collections on accounts receivable from a first entity.

B A derivative instrument is an instrument or other contract that has the following three characteristics: 1) at least one underlying and at least one notational amount or payment provision or both, 2) requires no initial net investment, or one that is smaller than would be required for other types of contracts expected to have a similar response to market factor changes, and 3) requires or permits net settlement, can be readily settled net by a means outside the contract, or provides for delivery of an asset that puts the recipient in a position not substantially different from net settlement.

A(n) ________________ is a number of currency units, shares, bushels, pounds, or other units specified in a derivative instrument. A Underlying B Notional amount C Firm commitment D Counter amount

B A notional amount is a number of currency units, shares, bushels, pounds, or other units specified in a derivative instrument.

PQR Ltd. enters into a contract with a customer for the sale of a tangible asset on January 1, 20X7, for $1 million. The contract also gives the entity the right to repurchase the asset for $1.1 million on or before December 31, 20X7. The expected market value for the asset is $1.05 million. This option is: A A forward option accounted for as a lease. B A call option accounted for as a financing arrangement. C A put option accounted for as a sale with a right to return. D A forward option accounted for as a financing arrangement.

B As PQR has the right to repurchase the asset this is a call option and as the repurchase price of 1.1 million is greater than the original price of $ 1 million, this is to be accounted for as a financing arrangement.

Neron Co. has two derivatives related to two different financial instruments, instrument A and instrument B, both of which are debt instruments. The derivative related to instrument A is a fair value hedge, and the derivative related to instrument B is a cash flow hedge. Neron experienced gains in the value of instruments A and B due to a change in interest rates. Which of the gains should be reported by Neron in its income statement? #Gain in value of debt instrument A Gain in value of debt instrument B A Yes/Yes B Yes/No C No/Yes D No/No

B The gain or loss of a fair value hedge (instrument A) is recognized in earnings in the period of change, together with the offsetting loss or gain in the hedged item. The effective portion of the gain or loss of a cash flow hedge (instrument B) is initially reported as a component of other comprehensive income (OCI) on the balance sheet, and subsequently reclassified into earnings when the forecasted transaction affects earnings.

A company incurred the following costs to complete a business combination in the current year: Issuing debt securities $30,000 Registering debt securities 25,000 Legal fees 10,000 Due diligence costs 1,000 What amount should be reported as current-year expenses, not subject to amortization? A $1,000 B $11,000 C $36,000 D $66,000

B) 10000+1000 = 11,000 Those expenses include -finder's fees -advisory legal, accounting, valuation, and other professional or consulting fees -general administrative costs Costs of registering and issuing debt and equity securities = Bond Issue Costs netted against the proceeds

Developing and publishing which of the following is not one of the stages in the due process the IASB uses is developing standards? A A Discussion Paper B A Comment Letter C An Exposure Draft D An IFRS

B) The development of comment letters is done by users in response to the IASB. The due process comprises the following six stages: 1. Research 2. Developing and publishing **the discussion paper** 3. Presenting a proposal 4. Developing and publishing the **Exposure Draft** 5. Developing and publishing the **IFRS** 6. Post-implementation review after the standard is issued.

A company recorded a decommissioning liability and recognized the amount recorded as part of the cost of the related property. After the property was fully depreciated, the decommissioning liability was reviewed and adjusted. How should this change in the decommissioning liability be recognized under IFRS? A The change in the liability is recognized in other comprehensive income. B The change in the liability is recognized in profit or loss. C The change in the liability is recognized as a change in the carrying amount of the property if the liability increases but is otherwise recognized in profit or loss. D The change in the decommissioning liability is not recognized until it is settled.

B) Under IFRS, changes in the decommissioning liability are added to or deducted from the cost of the asset, with any reductions in excess of the carrying amount of the asset recognized as a gain in the current period. The change in the liability is therefore recognized in profit or loss, not in other comprehensive income or as a change in the carrying amount of the property.

An entity may designate, as a type of hedge of foreign currency exposure, a derivative instrument or a non derivative financial instrument that may give rise to a foreign currency transaction gain or loss as which of the following? A A fair value hedge B A cash flow hedge C A hedge of a net investment in a foreign operation D None of the above

C An entity may designate a derivative instrument or a non derivative financial instrument that may give rise to a foreign currency transaction gain or loss as a hedge of the foreign currency exposure of a net investment in a foreign operation.

On December 1 of the current year, Bann Co. entered into an option contract to purchase 2,000 shares of Norta Co. stock for $40 per share (the same as the current market price) by the end of the next two months. The time value of the option contract is $600. At the end of December, Norta's stock was selling for $43, and the time value of the option is now $400. If Bann does not exercise its option until January of the subsequent year, which of the following changes would reflect the proper accounting treatment for this transaction on Bann's December 31, year-end financial statements? A The option value will be disclosed in the footnotes only. B Other comprehensive income will increase by $6,000. C Net income will increase by $5,800. D Current assets will decrease by $200.

C An option contract is a derivative. Derivatives are recognized as assets or liabilities on the financial statements and measures using fair value. Changes in fair value of non-hedge securities are reported as gains or losses in earnings. On December 1, Bann would initially record the option contract at $80,600 (2,000 shares × $40 market price on December 1 + $600 time value). At the end of December, the fair value of the option contract was $86,400 (2,000 shares × $43 market price on December 31 + $400 time value). Bann would report this change in fair value which would result in net income increasing by $5,800.

During the year, Pitt Corp. incurred costs to develop and produce a routine, low-risk computer software product, as follows: Design of tools, jigs, molds, and dies involving new technology $125,000 Completion of detail program design 13,000 Costs incurred for coding and testing to establish technological feasibility 10,000 Other coding costs after establishment of technological feasibility 24,000 Other testing costs after establishment of technological feasibility 20,000 Costs of producing product masters for training materials 15,000 Duplication of computer software and training materials From product masters (1,000 units)25,000 Packaging product (500 units) 9,000 In Pitt's December 31 Balance Sheet, what amount should be capitalized as software costs, subject to amortization? A $54,000 B $57,000 C $59,000 D $69,000

C (Testing$20,000+Coding24,000+producing15,000 = $ 59,000) Research and Development (R&D) costs are expensed. The completion of a detailed program design for $13,000 and costs incurred for coding and testing to establish technological feasibility of $10,000 are expensed. Costs incurred after software sales begin are Inventoried (include the duplication of computer software and training materials from product masters for $25,000 and packaging products for $9,000. The year-end balance sheet would report an inventory of $34,000.

A transaction was reported as a nonmonetary exchange of assets. Under which of the following circumstances should the exchange be measured based on the reported amount of the nonmonetary asset surrendered? A When the entity's future cash flows are expected to change as a result of the exchange. B When the timing of future cash flows of the asset received differs significantly from the configuration of the future cash flows of the asset transferred. C When the transaction lacks commercial substance. D When the transaction has commercial substance.

C When the transaction lacks commercial substance. Both exchanges and nonreciprocal transfers that involve little or no monetary assets or liabilities are referred to as nonmonetary transactions. Nonmonetary exchanges should be based on recorded amounts (rather than fair values) of the exchanged assets if any of the following conditions apply: (a) neither the fair value of the assets received nor the fair value of the assets surrendered is determinable within reasonable limits; (b) the transaction is an exchange of a product or property held for sale in the ordinary course of business for a product or property to be sold in the same line of business to facilitate sales to customers (other than the parties to the exchange); or (c) the transaction lacks commercial substance. In a nonmonetary exchange that has no commercial substance, the assets exchanged are accounted for at book value (after reduction, if appropriate, for an indicated impairment of value) of the nonmonetary asset(s) given up.

How should the acquirer recognize a bargain purchase in a business acquisition? A As negative goodwill in the statement of financial position B As goodwill in the statement of financial position C As a gain in earnings at the acquisition date D As a deferred gain that is amortized into earnings over the estimated future periods benefited

C) A bargain purchase is a business combination in which the FV of the recognized identifiable net assets acquired exceeds the FV of the acquirer's interest in the acquiree plus the recognized amount of any noncontrolling interest in the acquiree. In such cases, the acquirer should reassess whether it has correctly identified all of the assets acquired and all of the liabilities assumed and should recognize any additional assets or liabilities that are noted in that review. The objective of the review is to ensure that the measurements appropriately reflect consideration of all available information as of the acquisition date. If the excess remains after reassessment, the acquirer will recognize the resulting gain in earnings on the acquisition date.

Appointment of the IFRS Foundation Trustees is approved by which of the following? A The IFRS Advisory Council B The IFRS Interpretations Committee C The IFRS Foundation Monitoring Board D The Accounting Standards Advisory Forum

C) Appointment of the IFRS Trustees is approved by the IFRS Foundation Monitoring Board. The IFRS Advisory Council is the formal @@advisory body to the IASB and the Trustees@@ The IFRS Interpretations Committee **reviews current IFRS accounting issues** that have arisen and provides authoritative guidance (IFRICs) on those issues. It has 14 voting members appointed by the Trustees of the IFRS Foundation for their technical ability. The Accounting Standards Advisory Forum is ++an advisory group to the IASB providing technical advice and feedback.++ Its members consist of national accounting standard-setters and regional bodies with an interest in financial reporting and are selected by the Trustees.

Which of the following items would most likely require a subsequent event adjustment to the financial statements for the year ended December 31, year 1? A Uninsured loss of inventories purchased in year 1 as a result of a flood in year 2 B Settlement of litigation in year 2 over an event that occurred in year 2 C Loss on an uncollectible trade receivable recorded in year 1 from a customer that declared bankruptcy in year 2 before the financial statements were issued D Proceeds from a capital stock issuance in year 2 which was being approved by the board of directors in year 1

C) Loss on an uncollectible trade receivable recorded in year 1 from a customer that declared bankruptcy in year 2 is a classic example of providing evidence about conditions that existed at the balance sheet date and thus it requires a financial statement adjustment. There are two types of subsequent events: recognized and non-recognized. The first type requires ADJ of the FIN STM because it provides evidence that existed at the B/S date. The second type of subsequent event provides evidence that did not exist at the B/S date and does not require ADJ of the FIN STM; however, if material in nature, the event(s) may require disclosure to keep the FIN STM from being misleading.

Nolan owns 100% of the capital stock of both Twill Corp. and Webb Corp. Twill purchases merchandise inventory from Webb at 140% of Webb's cost. During the current year, merchandise that cost Webb $40,000 was sold to Twill. Twill sold all of this merchandise to unrelated customers for $81,200 during this year. In preparing combined financial statements for the year, Nolan's bookkeeper disregarded the common ownership of Twill and Webb. By what amount was unadjusted revenue overstated in the combined income statement for the year? A $16,000 B $40,000 C $56,000 D $81,200

C) Since the intercompany sale was not eliminated in preparing the combined financial statements, unadjusted revenue is overstated by $56,000 ($40,000 × 140%).

Zero Corp. suffered a material loss on an uncollectible trade account receivable after a customer suddenly went out of business due to a natural disaster ten days after Zero's balance sheet date, but one month before the issuance of the financial statements. Under these circumstances, the financial statements: A Should be adjusted, but should not disclose the event B Should be adjusted and should disclose the event C Should not be adjusted and should not disclose the event D Should not be adjusted, but should disclose the event

D

Initial Direct Cost (Finder's fee) -Amortization Annual Executory Cost (Maintenance) - Expense

Initial Direct Cost Dr. Lease Property 108,260 (105,260+3000) Cr. Lease Liability 105260 Cr. Cash 3000 Annual Executory Dr. Lease Liability 15000 Dr. Executory Cost 2000 Cr. Cash 17000

Historically, where separate incorporation is maintained, the subsidiary's financial records are which of the following? A Not affected by either the acquisition or the consolidation B Affected by the acquisition, but not the consolidation C Not affected by the acquisition, but affected by the consolidation D Affected by the acquisition and the consolidation

A)Historically, where separate incorporation is maintained, the subsidiary's financial records are not affected by either the acquisition or the consolidation.

On June 19, Don Co., a U.S. company, sold and delivered merchandise on a 30-day account to Cologne GmbH, a German corporation, for 200,000 euros. On July 19, Cologne paid Don in full. Relevant currency exchange rates were:June 19July 19Spot rate$ .988$ .99530-day forward rate.9901.000What amount should Don record on June 19 as an account receivable for its sale to Cologne? A $197,600 B $198,000 C $199,000 D $200,000

At the date a transaction is recognized, each asset, liability, revenue, expense, gain, or loss arising from a foreign currency transaction should be measured and recorded in the functional currency of the recording entity by use of the exchange rate (i.e. spot rate) in effect at that date. Don Co should record an account receivable of $197,600 on June 19 (200,000 x .988)

An entity is required to account for each business combination by applying the acquisition method. In applying the acquisition method, which of the following is *not required? A Determining the acquisition date B Recognizing and measuring identifiable assets acquired, liabilities assumed, and any noncontrolling interest in the acquiree C Allocating the excess of fair value over purchase price as a pro rata reduction of acquired assets D Recognizing and measuring goodwill or a gain from a bargain purchase

C) Applying the acquisition method requires: (1) identifying the acquirer, (2) determining the acquisition date, (3) recognizing and measuring identifiable assets acquired, liabilities assumed (4) recognizing and measuring goodwill or a gain from a bargain purchase.

Combined statements may be used to present the results of operations of Companies under common management / Commonly controlled companies A Yes/No B No/Yes C Yes/Yes D No/No

C) Combined financial statements are used if consolidated financial statements are not suitable for the same purpose. This is done when a non-consolidated subsidiary or group of companies owned by a common shareholder is present.

Measurement method for fair value

1. Identical asset in active markets 2. Similar asset in active markets 3. Unobservable inputs

Evaluating an asset for FV: The highest and best use of asset are:

1. Physically possible-Shape 2. Legally permissible-Law 3. Financial feasible-Income

During the current year, Orca Corp. decided to change from the FIFO method of inventory valuation to the weighted-average method. Inventory balances under each method were as follows: FIFO Weighted-average January 1 $71,000 $77,000 December 31 79,000 83,000 Orca's income tax rate is 30%. Orca should report the cumulative effect of this accounting change as a(an) A Prior period adjustment to the beginning balance of retained earnings B Component of income from continuing operations. C Extraordinary item. D Component of income after extraordinary items.

A A change in inventory valuation method is a change in accounting principle, and the cumulative effect of **the change in accounting principle should be reported as an adjustment to the beginning balance of retained earnings in the period of change.** It is not reported in the income statement.

The following information pertains to Flint Co.'s sale of 10,000 foreign currency units under a forward contract dated November 1, of the current year for delivery on January 31 of the following year: 11/1 12/31 Spot-rate $0.80 $0.83 30-day future rates 0.79 0.82 90-day future rates 0.78 0.81 Flint entered into the forward contract in order to speculate in the foreign currency. In Flint's income statement for the current year ended December 31, what amount of loss should be reported from this forward contract? A $400 B $300 C $200 D $0

A An agreement to exchange different currencies at a specified future date and at a specified rate (the forward rate). A forward contract is a foreign currency transaction. Therefore, a gain or loss on a forward contract is included in determining net income. Foreign currency units 10,000 Times: excess of forward rate available for the remaining maturity of the contract and the contracted forward rate ($0.82 - $0.78)× $0.04 Loss on forward contract $ 400

Which of the following items are not eligible to be measured using the fair value option? A Financial instruments that are classified by the user as a component of shareholder's equity B A firm commitment that would otherwise not be recognized at inception and only involves financial instruments C The rights and obligations under a warranty that is not a financial instrument but permits the warrantor to settle by paying a third party to provide goods or services D A host financial instrument resulting from the separation of an embedded nonfinancial instrument from a nonfinancial hybrid instrument

A Financial instruments that are classified by the user as a component of shareholder's equity are not eligible to be measured at fair value using the fair value option. Certain commitments, rights and obligations, and a host financial instrument resulting from the separation of an embedded nonfinancial instrument from a nonfinancial hybrid instrument are eligible to be measured using the fair value option.

The functional currency of Nash, Inc.'s subsidiary is the euro. Nash borrowed euros as a partial hedge of its investment in the subsidiary. In preparing consolidated financial statements, Nash's translation loss on its investment in the subsidiary exceeded its exchange gain on the borrowing. How should the effects of the loss and gain be reported in Nash's consolidated financial statements? A The translation loss less the exchange gain is reported in other comprehensive income. B The translation loss less the exchange gain is reported in net income. C The translation loss is reported in other comprehensive income and the exchange gain is reported in net income. D The translation loss is reported in net income and the exchange gain is reported in other comprehensive income.

A Translation adjustments are not be included in determining net income, but in OCI. Gains and losses on foreign currency transactions that are designated as, and are effective as, economic hedges of a net investment in a foreign entity are not included in determining net income, but are reported in the same manner as translation adjustments. Therefore, the net translation loss is be reported in OCI in Nash's consolidated financial statements.

On January 1, year 3, Pell Corp. purchased a machine having an estimated useful life of 10 years and no salvage. The machine was depreciated by the double declining balance method for both financial statement and income tax reporting. On January 1, year 8, Pell changed to the straight-line method for financial statement reporting but not for income tax reporting. Accumulated depreciation at December 31, year 7, was $560,000. If the straight-line method had been used, the accumulated depreciation at December 31, year 7, would have been $420,000. Pell's enacted income tax rate for year 8 and thereafter is 30%. The amount shown in the year 8 income statement for depreciation expense after changing to the straight-line method should be a A $56,000 debit. B $84,000 debit. C $84,000 credit. D $98,000 credit.

A change in depreciation method, such as from the double declining balance to the straight-line method is a change in accounting principle that is inseparable from a change in accounting estimate. When the effects of the two changes cannot be separated, the change should be treated as a change in estimate. A change in accounting estimate should be accounted for in the *period of change if the change only effects that period, or in the current and subsequent periods*, if the change effects both, as a component of income from continuing operations. If accumulated depreciation using the straight-line method at the end of year 7, 5 years worth, would have been $420,000 the machine must have cost $840,000. The $840,000 less accumulated depreciation of $560,000 gave the machine a $280,000 carrying value as of January 1, year 8. Dividing that $280,000 by 5 more remaining straight-line years provides for $56,000 of depreciation expense in each of the remaining years. Depreciation expense would be debited for $56,000. *No prior period adjustment is made.*

Fogg Co., a U.S. company, contracted to purchase foreign goods. Payment in foreign currency was due one month after the goods were received at Fogg's warehouse. Between the receipt of goods and the time of payment, the exchange rates changed in Fogg's favor. The resulting gain should be included in Fogg's financial statements as a(an) A Component of income from continuing operations. B Extraordinary item. C Deferred credit. D Item of other comprehensive income.

A change in exchange rates between the functional currency and the currency in which the transaction is denominated increases or decreases the expected amount of functional currency cash flows upon a settlement of the transaction. That increase or decrease in expected functional currency cash flows is a foreign currency transaction gain or loss that generally should be included as a component of income from continuing operations for the period in which the transaction is settled.

Which of the following criteria must be met for a lease to be classified as a direct financing lease? A The lease term is for the major part of the remaining economic life of the underlying asset. However, if the commencement date falls at or near the end of the economic life of the underlying asset, this criterion shall not be used for purposes of classifying the lease. B The present value of the sum of the lease payments and any residual value guaranteed by the lessee that is not already reflected in the lease payments equals or exceeds substantially all of the fair value of the underlying asset. C The underlying asset is of such a specialized nature that it is expected to have no alternative use to the lessor at the end of the lease term. D The present value of the sum of the lease payments and any residual value guaranteed by the lessee that is not already reflected in the lease payments and/ or any other third party unrelated to the lessor equals or exceeds substantially all of the fair value of the underlying asset.

A lessor shall classify the lease as either a direct financing lease or an operating lease. A lessor shall classify the lease as an operating lease unless *both of the following criteria are met, in which case the lessor shall classify the lease as a direct financing lease*: 1. The PV of the sum of the lease payments and any residual value guaranteed by the lessee that is not already reflected in the lease payments and/ or any other third party unrelated to the lessor equals or exceeds substantially all of the fair value of the underlying asset. 2.It is probable that the lessor will collect the lease payments plus any amount necessary to satisfy a residual value guarantee.

Included within the financial assets of Z square Co. at December 31, 2017 are the following two recently purchased investments in publicly-traded (listed in NASDAQ) equity shares: Investment 1: 10% of the issued share capital of ABC Co. This shareholding was acquired as a long-term investment as Z square Co. wishes to participate as an active shareholder of ABC Co. Investment 2: 10% of the issued share capital of XYZ Co. This shareholding was acquired for speculative purposes and Z square Co. expects to sell these shares in the near future. Neither of these shareholdings gives Z square Co. significant influence over the investee companies. Wherever possible, the directors of Z square Co. wish to avoid taking any fair value movements to income statement, so as to minimize volatility in reported earnings. How should the fair value movements in these investments be reported in Z square Co's financial statements for the year ended December 31, 2017? A In income statement for both investments B In other comprehensive income for both investments C In income statement for investment 1 and in other comprehensive income for investment 2 D In other comprehensive income for investment 1 and in income statement for investment 2

A) As the shares of both the companies ABC Co. and XYZ Co. are listed in NASDAQ, thus, the fair value of both investments is readily determinable and for any movement in fair value should be reported in the income statement as Fair Value Through Net Income (FVTNI). Options (B), (C) and (D) are incorrect because for movement in fair value for investment in both securities should be reported in income statement only and not through other comprehensive income.

A holder of a variable interest that is not the primary beneficiary acquired additional variable interests in the variable interest entity (VIE). What action, if any, should follow? A The holder of the variable interest should reconsider whether it is now the primary beneficiary. B The holder of the variable interest should use the voting-interest model to determine whether the VIE should be consolidated. C The primary beneficiary should discontinue consolidation of the VIE because the election to consolidate is no longer allowed. D No action is necessary because the primary beneficiary of a VIE does not change subsequent to the initial assessment.

A) Find a new one A variable interest entity (VIE) is a legal entity subject to consolidation according to GAAP. An entity is the primary beneficiary when it has the power to direct activities and it has the obligation to absorb loss/benefits from the VIE.

Shore Co. records its transactions in U.S. dollars. A sale of goods resulted in a receivable denominated in Japanese yen, and a purchase of goods resulted in a payable denominated in euros. Shore recorded **a foreign exchange gain on collection of the receivable and an exchange loss on settlement of the payable**. The exchange rates are expressed as so many units of foreign currency to one dollar. Did the number of foreign currency units exchangeable for a dollar increase or decrease between the contract and settlement dates? #Yen exchangable for $1/Euros exchangable for $1 A Increase/Increase B Decrease/Decrease C Decrease/Increase D Increase/Decrease

B To record a foreign exchange gain on collection of the receivable, the exchange rate of Japanese yen to one U.S. dollar must have decreased. For instance, assume a receivable for l,000 yen and an exchange rate of 10 yen to 1 dollar at the contract date. If the exchange rate changed to 5 yen to 1 dollar, the 1,000 yen when collected could be converted into 200 U.S. dollars at the settlement date. This $200 is twice the $100 that could have been obtained from the conversion of currency at the contract date (1,000 / 10 = $100). To record an exchange loss on settlement of the payable, the exchange rate of euros to one dollar must have decreased. For example, assume a payable for 1,000 euros and an exchange rate of 4 euros to 1 dollar at the contract date. If the exchange rate changed to 2 euros to 1 dollar, the 1,000 euross paid at the settlement date required more U.S. dollars than would have been required at the date of contract (1,000 / 2 = $500 at date of settlement versus 1,000 / 4 = $250 at date of contract).

Giaconda, Inc. acquires an asset for which it will measure the fair value by discounting future cash flows of the asset. Which of the following terms best describes this fair value measurement approach? A Market B Income C Cost D Observable inputs

B Using present value techniques to discount the cash flows or earnings is called the income approach. Option (A) is incorrect because **market approach uses prices and relevant information from market transactions for identical or comparable assets/liabilities.** Option (C) is incorrect because **cost approach uses current replacement cost.** Option (D) is incorrect because **observable inputs are the inputs for FV valuation and not a valuation technique.**

Which of the following is the market with the greatest volume or level of activity for an orderly transaction to occur for an asset or liability? A Perfect market B Principal market C Financial market D Most advantageous market

B - The principle market = greatest volume / level of activity for an asset or liability = FV measurement A perfect market = perfect info and perfect competition. A financial market = easily buy and sell financial securities. Accounting standards make no reference to perfect or financial markets. A most advantageous market is the market with the price that maximizes the amount to transfer a liability.

Mr. and Mrs. Gasson own 100% of the common stock of Able Corp. and 90% of the common stock of Baker Corp. Able previously paid $4,000 for the remaining 10% interest in Baker. The condensed December 31 balance sheets of Able and Baker are as follows: Able Baker Assets $600,000 $60,000 Liabilities 200,000 30,000 Common stock 100,000 20,000 Retained earnings 300,000 10,000 $600,000 $60,000 In a combined balance sheet of the two corporations at December 31, what amount should be reported as total stockholders' equity? A $430,000 B $426,000 C $403,000 D $400,000

B) Common stock ($100,000 + $20,000) $120,000 Retained earnings ($300,000 + $10,000) 310,000 Subtotal430,000 Less Able's Paid 10% interest in Baker (4,000) Stockholders' equity in combined balance sheet $426,000

A company incurred the following costs to complete a business combination in the current year: Legal Fees - the issue of debt securities $30,000 Cost of Registering debt securities $25,000 Legal fees related to Business Combination $10,000 Due diligence costs $1,000 What amount should be reported as Bond Issue Costs? A $1,000 B $55,000 C $36,000 D $66,000

B) Bond Issue Costs includes costs incurred in preparing and selling a bond issue: Legal fees Accounting fees Underwriting Commissions Registration, Printing, and Engraving

During December of the previous year, Nile Co. incurred special insurance costs but did not record these costs until payment was made during the current year. These insurance costs related to inventory that had been sold by December 31 of the previous year. What is the effect of the omission on Nile's accrued liabilities and retained earnings at December 31 of the previous year? #Accrued liabilities/Retained earnings A No effect/No effect B No effect/Overstated C Understated/Overstated D Understated/No effect

C Accrual accounting recognized and reports the effects of transactions and other events on the assets and liabilities of a business enterprise in the time periods to which they relate rather than only when cash is received or paid. Accrual accounting attempts to match revenues and the expenses associated with those revenues in order to determine net income for an accounting period. The insurance costs were not recorded during the period to which they relate, so the accrued liabilities is understated. Because the expense was not included in the inventory that was sold, the **COGS was less than it should have been, and as a result, the retained earnings were overstated**.

DeeCee Co. adjusted its historical cost income statement by applying specific price indexes to its depreciation expense and cost of goods sold. DeeCee's adjusted income statement is prepared according to A Fair value accounting. B General purchasing power accounting. C Current cost accounting. D Current cost/general purchasing power accounting.

C DeeCee adjusts the depreciation and cost of goods sold reported in the historical cost income statement by applying specific price indexes to these amounts. Therefore, DeeCee's adjusted income statement is prepared using current cost accounting. The income statement is not prepared using fair value accounting because only depreciation expense and cost of goods sold are restated by applying specific price indexes. The income statement is not prepared using general purchasing power accounting because DeeCee's historical costs are not remeasured into units of a currency with the same general purchasing power. The income statement is not prepared using current cost/general purchasing power accounting because amounts are not remeasured into units of a currency with the same general purchasing power.

Which of the following items requires a prior period adjustment to retained earnings? A Purchases of inventory this year were overstated by $5 million. B Available-for-sale securities were improperly valued last year by $20 million. C Revenue of $5 million that should have been deferred was recorded in the previous year as earned. D The prior year's foreign currency translation gain of $2 million was never recorded.

C Errors in financial statements result from mathematical mistakes, mistakes in the application of accounting principles, or the oversight or misuse of facts that existed at the time the financial statements were prepared. Errors that occur in one accounting period and are discovered in a subsequent accounting period are more involved: the cumulative effect of each error on periods prior to the period of discovery is calculated and recorded as a direct adjustment to the beginning balance of retained earnings. Errors that occur and are discovered in the same accounting period may be corrected by reversing the incorrect entry and recording the correct one or by directly correcting the account balances with a single entry. *Foreign currency gains and losses and available for sale securities are reported in other comprehensive income.*

A company owns a financial asset that is actively traded on two different exchanges (market A and market B). There is no principal market for the financial asset. The information on the two exchanges is as follows: Quoted price of asset /Transaction costs Market A $ 1,000 $ 75 Market B 1,050 150 What is the fair value of the financial asset? A $ 900 B $ 925 C $1,000 D $1,050

C Fair value is the price that would be received to sell an asset or paid to transfer a liability in an orderly transaction between market participants at the measurement date. It is assumed that the transaction would occur in the principle market, or in the absence of a principle market, the most advantageous market would be used. The most advantageous market is the market with the price that maximizes the amount that would be received for the asset or minimizes the amount that would be paid to transfer the liability. In determining the most advantageous market, transaction costs are netted against the price of the asset. The fair value price is not adjusted for transaction costs. Market A would yield a net $925 ($1,000 - $75) Market B would yield $900 ($1,050-$150) Market A is the most advantageous, and the fair value of the asset would be $1,000.

In sales-type leases, the lessor must disclose which of the following? A The cost and carrying amount, if different, of property leased or held for leasing, by major class and total accumulated depreciation B A general description of leasing arrangements C Net investment components D All of the above

C In sales-type and direct financing leases, the lessor must disclose the net investment components, including: future MLP; unguaranteed residual value; unearned income; and the future MLP to be received in each of the succeeding 5 years. For operating leases, the lessor must disclose: the cost and carrying amount, if different, of property leased or held for leasing, by major class and total accumulated depreciation; the minimum future rentals on noncancelable leases, in aggregate, for each of the next 5 years; and a general description of leasing arrangements.

Which of the following is not a part of Lease Payments? A Variable lease payments dependent on a rate or index B Payments associated with renewal or termination options or the exercise of a purchase option C Variable lease payments not dependent on an index or a rate D Fees Paid by the Lessee to Owners of Special-Purpose Entities

C Lease payments include: 1. Fixed Lease Payments and in substance fixed payments 2. Variable lease payments dependent on a rate or index. 3. Payments associated with renewal or termination options or the exercise of a purchase option 4. Fees Paid by the Lessee to Owners of Special-Purpose Entities 5. Residual Value Guarantee Lease payments exclude: 1. Variable lease payments not dependent on an index or a rate 2. Guarantee of lessors debt

Each of the following would be considered a Level 2 observable input that could be used to determine an asset or liability's fair value, except A Quoted prices for identical assets and liabilities in markets that are not active. B Quoted prices for similar assets and liabilities in markets that are active. C Internally generated cash flow projections for a related asset or liability. D Interest rates that are observable at commonly quoted intervals.

C Level 2 inputs are directly or indirectly observable inputs; it includes: Quoted prices from active markets for similar assets/liabilities Quoted prices from limited activity markets for similar/identical assets Other observable inputs like yield curves, bank prime rates, interest rates, credit risks, default rates on loans.

Compared to its year 2 cash basis net income, Potoma Co.'s year 2 accrual basis net income increased when it A Declared a cash dividend in year 1 that it paid in year 2. B Wrote off more accounts receivable balances than it reported as uncollectible accounts expense in year 2. C Had lower accrued expenses on December 31, year 2, than on January 1, year 2. D Sold used equipment for cash at a gain in year 2.

C Potoma's accrued expenses decreased during year 2. Hence, Potoma's year 2 payments for expenses exceeded the amount of expense recognized on the accrual basis in year 2. The increased amount of expenses recognized in year 2 under the cash basis increases Potoma's year 2 accrual basis net income as compared to its year 2 cash basis net income. The declaration or payment of a cash dividend does not affect net income computed under either the cash or accrual basis. Compared to its year 2 cash basis net income, Potoma's year 2 accrual basis net income decreased when it recognized uncollectible accounts expense in year 2. Potoma's year 2 cash basis net income is not affected by either the accounts receivable balances written off in year 2 or the uncollectible account expense recognized in year 2. The sale of the used equipment at a gain increases net income under both the cash and accrual basis by equal amounts.

The fair value for an asset or liability is measured as A The appraised value of the asset or liability. B The price that would be paid to acquire the asset or received to assume the liability in any type of transaction between two parties. C The price that would be received when selling an asset or paid when transferring a liability in an orderly transaction between market participants. D The cost of the asset less any accumulated depreciation or the carrying value of the liability on the date of the sale.

C The fair value is defined as the price that would be received to sell an asset or paid to transfer a liability in an orderly arm's length transaction between market participants at the measurement date (at exit price). Option (A) is incorrect because the appraised value is not the Fair Value (FV). Option (B) is incorrect because FV is the exit price and not the entry price. Option (D) is incorrect because the carrying value is the historical cost and not the FV.

On January 1, year 1, West Co. entered into a 10-year lease for a manufacturing plant. The annual minimum lease payments are $100,000. In the notes to the December 31, year 2 financial statements, what amounts of subsequent years' lease payments should be disclosed? #Amount for required period / #Aggregate amount for period thereafter A $100,000/$0 B $300,000/$500,000 C $500,000/$300,000 D $500,000/$0

C The notes to the financial statements need to disclose the amount of minimum lease payments due for each of the next five years and the aggregate amount for the period thereafter and the amount totally in aggregate. The amount for the required period = Amount for 5 years. West discloses the minimum lease payments (MLP) of $500,000 ($100,000 × 5) pertaining to the next five succeeding fiscal years and the $300,000 [$100,000 × (10 - 2 - 5)] of MLP pertaining to the fiscal years after this 5-year period. Lease amounts in aggregate is 800,000 (i.e. $80,000 x 8)

The primary purpose of a quasi-reorganization is to give a corporation the opportunity to A Obtain relief from its creditors. B Revalue understated assets to their fair values. C Eliminate a deficit in retained earnings. D Distribute the stock of a newly-created subsidiary to its stockholders in exchange for part of their stock in the corporation.

C The primary purpose of a quasi-reorganization is to eliminate an accumulated deficit (negative retained earnings balance) so that the corporation has a "fresh start" with a zero balance in retained earnings. Although the accounting procedures for a quasi-reorganization involve restating assets of the enterprise to their fair values, this is not the primary purpose.

While preparing its year 3 financial statements, Dek Corp. discovered computational errors in its year 2 and year 1 depreciation expense. These errors resulted in overstatement of each year's income by $25,000, net of income taxes. The following amounts were reported in the previously issued financial statements: Year 2 Year 1 Retained earnings, 1/1 $700,000 $500,000 Net income 150,000 200,000 Retained earnings, 12/31 $850,000 $700,000 Dek's year 3 net income is correctly reported at $180,000. Which of the following amounts should be reported as prior period adjustments and net income in Dek's year 3 and year 2 comparative financial statements? #YearPrior period adjustment/Net income A 2 ---- 150,000 3 $(50,000) 180,000 B 2 $(50,000) 150,000 3 --- 180,000 C 2 $(25,000) 125,000 3 --- 180,000 D 2 --- 125,000 --- 180,000

C The understatement of year 1 and year 2 depreciation expense discovered in preparing the year 3 financial statements represents the correction of an error in previously issued financial statements (i.e., a prior period adjustment). Prior period adjustments are reported retroactively by (1) correcting all prior period statements presented and (2) restating the beginning balance of retained earnings for the first period presented when the error effects extend to a period prior to that one. Therefore, since year 3 and year 2 comparative financial statements are presented, the understatement of year 1 depreciation expense should be reported as a prior period adjustment in the year 2 financial statements as a $25,000 decrease to the beginning balance of retained earnings. In addition, year 2 net income should be reported at $125,000 (i.e., $150,000 - $25,000) in order to reflect the correct amount of year 2 depreciation expense. Since the understatement of depreciation expense pertains to year 1 and year 2 and is reported retroactively, the reported amount of year 3 net income of $180,000 is not affected by the errors.

Under IFRS 9, an entity's financial assets can be classified and measured in which ways? A Held to Maturity, Available-for-sale and Trading B Fair Value through Profit of Loss and Amortized Cost C Fair Value through Profit or Loss, Amortized Cost and Fair Value through Other Comprehensive Income D Available-for-Sale and Amortized Cost

C Under IFRS 9, the default classification of financial assets is **Fair Value through Profit or Loss' (FVTPL). FVTPL includes any financial assets that are held for trading and are also derivatives. Changes in fair value are reported through profit or loss at each reporting date.** A second classification of financial assets is **Fair Value through Other Comprehensive Income (FVTOCI). Changes in fair value are reported through other comprehensive income at each reporting date.** Finally the third classification that can be measured under IFRS is **amortized cost.**

In its financial statements, Pulham Corp. uses the equity method of accounting for its 30% ownership of Angles Corp. At December 31 of the current year, Pulham has a receivable from Angles. How should the receivable be reported in Pulham's year-end financial statements? A None of the receivable should be reported, but the entire receivable should be offset against Angles' payable to Pulham. B Seventy percent of the receivable should be separately reported, with the balance offset against 30% of Angles' payable to Pulham. C The total receivable should be disclosed separately. D The total receivable should be included as part of the investment in Angles, without separate disclosure.

C) 30% is not controlling interest (i.e., > 50%). Therefore, there is no reason to treat the separate entities as one.

n January 1, year 1, a company capitalized $100,000 of costs for software that is to be sold. The company amortizes the software costs on a straight-line basis over five years. The carrying value of the software costs on January 1, year 3, was $60,000. As of December 31, year 3, the estimated future gross revenue to be generated from the sale of the software is $23,000, and the estimated future cost of disposing of the software is $8,000. What amount should the company expense related to the software costs for the year ended December 31, year 3? A $18,400 B $20,000 C $37,000 D $45,000

Capitalized software costs are amortized on a product-by-product basis. The annual amortization is the greater of the amount computed using (1) the ratio that current gross revenues for a software product bear to the total of current and anticipated future gross revenue for that product, or (2) the straight-line method over the remaining estimated economic life of the product, including the current period being reported on. Amortization shall be based on estimated future revenues. Because of the uncertainties involved in estimating revenue, amortization shall not be less than straight-line amortization over the product's remaining estimated economic life. The company would add the future cost of disposing the software to the carrying value of the software costs, then bring those software costs to be amortized down to the $23,000 estimated future gross revenue amount and expense the rest ($60,000 + $8,000 - $23,000 = $45,000).

Which of the following is a true statement related to foreign currency accounting and reporting under US GAAP and IFRS? A Only IFRS requires the identification of hyperinflationary economies. B Both US GAAP and IFRS require foreign currency transactions to be re-measured into an entity's functional currency with amounts resulting from changes in exchange rates being reported in other comprehensive income. C Both US GAAP and IFRS require the "step-by-step" method of consolidation in reporting the consol-idation of foreign operations. D Both US GAAP and IFRS require re-measurement into the functional currency before translation into the reporting currency.

D **Both US GAAP and IFRS require re-measurement into the functional currency before translation into the reporting currency. Assets and liabilities are translated at the period-end rate and income statement amounts generally are translated at the average rate, with the exchange differences reported in equity. Both **US GAAP and IFRS also require the identification of hyperinflationary economies and foreign currency transactions to be re-measured into an entity's functional currency with amounts resulting from changes in exchange rates being reported in net income, not other comprehensive income. **Under US GAAP, the "step-by-step" method of consolidation is used when reporting the consolidation of foreign operations, and **under IFRS, either the "direct" or the "step-by-step" method of consolidation may be used.

Compared to the accrual basis of accounting, the cash basis of accounting understates income by the net decrease during the accounting period of Accounts receivable Accrued expenses A Yes Yes B Yes No C No No D No Yes

D **When accounts receivable decrease during a period, revenues on a cash basis are more than revenues on an accrual basis because cash collections are more than revenues reported on an accrual basis. Thus, compared to the accrual basis of accounting, the cash basis of accounting overstates income by the net decrease in A/R.** ++When accrued expenses decrease during a period, expenses on a cash basis are more than expenses on an accrual basis because cash payments are more than expenses reported on an accrual basis. Thus, compared to the accrual basis of accounting, the cash basis understates income by the net decrease in the accrued expenses.++

At December 31 of the current year, Off-Line Co. changed its method of accounting for demo costs from writing off the costs over two years to expensing the costs immediately. Off-Line made the change in recognition of an increasing number of demos placed with customers that did not result in sales. Off-Line had deferred demo costs of $500,000 at December 31 of the previous year, $300,000 of which were to be written off in the current year and the remainder in the following year. Off-Line's income tax rate is 30%. In its current year financial statements, what should Off-Line report? A $140,000 as cumulative effect of change in accounting principle B $200,000 as current period transaction with no prior period adjustment C $350,000 as cumulative effect of change in accounting principle D $500,000 as current period transaction with no prior period adjustment

D A change in accounting estimate affected by a change in accounting principle is accounted for as a change in estimate. A change in accounting estimate shall be accounted for in (a) the period of change if the change affects that period only or (b) the period of change and future periods if the change affects both. A change in accounting estimate shall not be accounted for by restating or retrospectively adjusting amounts reported in financial statements of prior periods or by reporting pro forma amounts for prior periods. Because the change was made, the total amount of $500,000 would be recorded.

On January 1, year 1, Warren Co. purchased a $600,000 machine, with a five-year useful life and no salvage value. The machine was depreciated by an accelerated method for book and tax purposes. The machine's carrying amount was $240,000 on December 31, year 2. On January 1, year 3, Warren changed retroactively to the straight-line method for financial statement purposes. Warren can justify the change. Warren's income tax rate is 30%. In its year 3 income statement, what amount should Warren report as a prior period adjustment as result of this change? A $120,000 B $ 84,000 C $ 36,000 D $0

D A change in depreciation method, such as from the accelerated to the straight-line method is a change in accounting principle that is inseparable from a change in accounting estimate. When the effects of the two changes cannot be separated, the change should be treated as a change in estimate. A change in accounting estimate should be accounted for in the period of change if the change only affects that period, or in the current and subsequent periods, if the change affects both, as a component of income from continuing operations. No prior period adjustment is made.

Pear Co.'s income statement for the current year ended December 31, as prepared by Pear's controller, reported income before taxes of $125,000. The auditor questioned the following amounts that had been included in income before taxes: Equity in earnings of Cinn Co.$40,000 Dividends received from Cinn 8,000 Adjustments to profits of prior years for arithmetical errors in depreciation (35,000) Pear owns 40% of Cinn's common stock. Pear's December 31 income statement should report income before taxes of A $ 85,000 B $117,000 C $120,000 D $152,000

D Since Pear owns 40% of Cinn's common stock, Pear has the ability to exercise significant influence over Cinn by virtue of its investment and should account for its investment in Cinn by the equity method. Therefore, Pear's $40,000 equity in Cinn's earnings is properly included in Pear's current year income before taxes. Under the equity method, the dividends received from Cinn reduce the carrying amount of the investment; they do not affect the amount of investment income that Pear recognizes. So, the $8,000 of dividends received from Cinn is erroneously included in current year income, before taxes are subtracted to correct that figure. The arithmetical errors in depreciation of prior years represents a correction of errors of prior periods. **The correction of the errors should be reported as a prior-period adjustment by restating the prior-period financial statements**. So, the $35,000 of arithmetical errors in depreciation of prior years that Pear had inadvertently subtracted from current year income before taxes are added back to correct that figure. Income before taxes, before adjustment $125,000 Less: Dividends received from equity method investee (8,000) Add: Arithmetical errors in depreciation of prior years _ 35,000 Corrected income before taxes $152,000

Which of the following errors could result in an overstatement of both current assets and stockholders' equity? A An understatement of accrued sales expenses. B Noncurrent note receivable principal is misclassified as a current asset. C Annual depreciation on manufacturing machinery is understated. D Holiday pay expense for administrative employees is misclassified as manufacturing overhead.

D Since holiday pay for administrative employees is a period cost, it should have been expensed when incurred. Instead, it was misclassified and **inventoried** as manufacturing overhead. This error overstates both inventory and net income, thus overstating both current assets and stockholder's equity. The understatement of an accrued expense overstates net income and stockholders' equity and understates current liabilities. Misclassifying noncurrent note receivable principal as a current asset overstates current assets and understates noncurrent assets. The understatement of depreciation expense overstates net income, noncurrent assets, and stockholders' equity.

Ball Corp. had the following foreign currency transactions during the current year:Merchandise was purchased from a foreign supplier on January 20 for the U.S. dollar equivalent of $90,000. The invoice was paid on March 20 at the U.S. dollar equivalent of $96,000.On July 1, Ball borrowed the U.S. dollar equivalent of $500,000 evidenced by a note that was payable in the lender's local currency on July 1 in two years. On December 31, the U.S. dollar equivalents of the principal amount and accrued interest were $520,000 and $26,000, respectively. Interest on the note is 10% per annum.In Ball's year-end income statement, what amount should be included as foreign exchange loss? A $0 B $ 6,000 C $21,000 D $27,000

D The payables resulting from the merchandise purchased from the foreign supplier and the borrowing are transactions denominated in a foreign currency. The payable from the merchandise purchased was recorded at $90,000 on 1/20. It was paid on 3/20 at the U.S. dollar equivalent of $96,000, resulting in a $6,000 foreign exchange loss. The payable from the borrowing was recorded at $500,000 at 7/1. Accrued interest on the borrowing was $25,000 ($500,000 × 10% × 6/12) at 12/31. At 12/31, the U.S. dollar equivalents of principal and accrued interest were $520,000 and $26,000, respectively, resulting in an additional $21,000 [($520,000 + $26,000) - ($500,000 + $25,000)] foreign exchange loss. Thus, the total amount of foreign exchange loss to be recognized in the yearend income statement is $27,000 ($6,000 + $21,000).

Company J acquired all of the outstanding common stock of Company K in exchange for cash. The acquisition price exceeds the fair value of net assets acquired. How should Company J determine the amounts to be reported for the plant and equipment and long-term debt acquired from Company K? Plant and equipment / Long-term debt A K's carrying amount/K's carrying amount B K's carrying amount/Fair value C Fair value/K's carrying amount D Fair value/Fair value

D - All Assets = FV @ acquisition If acquisition price > FV = goodwill

On January 1, year 1, Peabody Co. purchased an investment for $400,000 that represented 30% of Newman Corp.'s outstanding voting stock. For year 1, Newman reported net income of $60,000 and paid dividends of $20,000. At year end, the fair value of Peabody's investment in Newman was $410,000. Peabody elected the fair value option for this investment. What amount should Peabody recognize in net income for year 1 attributable to the investment? A $6,000 B $10,000 C $16,000 D $18,000

Entities may choose to measure eligible items at fair value (the "fair value option") that are not currently required to be measured at fair value. The decision to elect the fair value option is applied instrument by instrument, is irrevocable, and is applied only to an entire instrument. A business entity shall report unrealized gains and losses on items for which the fair value option has been elected in earnings at each subsequent reporting date. The Investment in Newman would be increased by 30% of the net income and decreased by 30% of the dividends, resulting in a year end carrying amount of $412,000 ($400,000 + 18,000 - 6,000). Since the fair value was $410,000, Peabody had an unrealized loss of $2,000. This loss is netted against the investment income previously recognized of $18,000 for a $16,000 net income impact. **Dividends do not affect net income** (they reduce the Investment account).

Matt Co. included a foreign subsidiary in its consolidated financial statements. The subsidiary was acquired six years ago and was excluded from previous consolidations. The change was caused by the elimination of foreign exchange controls. Including the subsidiary in the consolidated financial statements results in accounting change that should be reported A By footnote disclosure only. B Currently and prospectively. C Currently with footnote disclosure of pro forma effects of retroactive application. D By retrospective application to the financial statements of all prior periods presented.

Since the subsidiary was excluded from previous consolidations, its inclusion in the current year's consolidated financial statements results in a change in accounting principle in which the financial statements, in effect, are those of a different reporting entity. The change in the reporting entity should be reported by retrospective application to the financial statements of all prior periods presented to reflect the new reporting entity.

Howe Co. leased equipment to Kew Corp. on January 2, year 1, for an eight-year period expiring December 31, year 8. Equal payments under the lease are $600,000 and are due annually on January 2. The first payment was made on January 2, year 1. The list selling price of the equipment is $3,520,000 and its carrying cost on Howe's books is $2,800,000. The lease is appropriately accounted for as a sales-type lease. The present value of the lease payments at an imputed interest rate of 12% (Howe's incremental borrowing rate) is $3,300,000. What amount of profit on the sale should Howe report for the year ended December 31, year 1? A $720,000 B $500,000 C $ 90,000 D $0

Profit on the sale should be reported for the excess of the present value of the minimum lease payments over the carrying amount of the equipment. The list price of the equipment is irrelevant. The normal sales price would only be used as the fair value if the present value of the lease payments wasn't given or able to be computed. The fair value of the leased property at the lease inception is the present value of the minimum lease payments at the lessor's incremental borrowing rate. The excess of the present value of the minimum lease payments of leased property at the lease inception over its cost or carrying amount is the dealer's profit from a sales-type lease and recognized fully at the lease inception. FV of leased property $3,300,000 Less:CarryValueequipment(2,800,000) Manufacturer's profit sale $ 500,000

Certain balance sheet accounts of a foreign subsidiary of Rowan Inc., at December 31, have been translated into U.S. dollars as follows: Translated at Current Rates Historical Rates Note receivable, long-term $240,000 $200,000 Prepaid rent 85,000 80,000 Patent 150,000 170,000 $475,000 $450,000 The subsidiary's functional currency is the currency of the country in which it is located. What total amount should be included in Rowan's December 31 consolidated balance sheet for the above accounts? A $450,000 B $455,000 C $475,000 D $495,000

Since the subsidiary's functional currency is the currency of the country in which it is located, all of its assets are translated at the current rate (i.e., the exchange rate in effect at the balance sheet date).

Rig Co. sold its factory at a gain, and simultaneously leased it back for 10 years. The factory's remaining economic life is 20 years. The lease was reported as an operating lease. At the time of sale, Rig should report the gain as: A In the income statement B An asset valuation allowance. C A separate component of stockholders' equity. D A deferred credit.

The correct answer is (A). FASB issued ASC 842 to amend accounting & reporting for leases under which for a sale to occur in the context of a sale and leaseback transaction, the transfer of the asset must meet the requirements for a sale per the Revenue Recognition standards. If there is no sale for the seller-lessee, the buyer-lessor also does not account for a purchase. Any consideration paid for the asset is accounted for as a financing transaction by both the seller-lessee and the buyer-lessor. If the leaseback is a finance lease from seller-lessee's perspective, transfer of the asset is not a Sale. However, here the leaseback is an operating lease. The transaction is recognized as a sale and the entire gain is recognized immediately on the income statement. Note: Criteria for Substantial Economic Interest 1. Term of Lease is a large % of remaining life. (OR) 2. If the PV of lease payment => than 90% FV asset.. If Yes 1 or 2, must CR deferred gain (can't put gain I/S)

On October 1 of the current year, Mild Co., a U.S. company, purchased machinery from Grund, a German company, with payment due on April 1 of next year. If Mild's current-year operating income included no foreign exchange transaction gain or loss, then the transaction could have A Resulted in a Foreign Currency translation gain to be reported in company's income statement. B Been denominated in U.S. dollars. C Caused a foreign currency gain to be reported as a contra account against machinery. D Caused a foreign currency translation gain to be reported in other comprehensive income

The correct answer is (B). Foreign currency transactions are transactions denominated in a currency other than the entity's functional currency. Hence, no foreign currency transaction gain or loss would occur if the purchase of the machinery by the U.S. company is denominated in U.S. dollars.

An investment company's portfolio of private placement securities is recorded at fair value and valued using a matrix pricing model. The matrix pricing model uses current pricing spreads on similar securities to determine the fair value of the private placement securities. Which of the following valuation techniques is being used? A The cost approach. B The market approach. C The exchange approach. D The income approach.

The correct answer is (B). Market Approach uses prices and other relevant information generated by market transactions involving identical or comparable assets or liabilities. In the case presented in this problem statement, current pricing spreads on similar securities are being used to determine the fair value of private placement securities. As such, the company is using the market approach valuation technique. Cost Approach is based on the amount that currently would be required to replace the service capacity of an asset, often referred to as the current replacement cost. Income Approach uses valuation techniques to convert future amounts to a single present amount.

The details of a share's price from two markets are given as under: Market Volume Sold Price Transaction Cost A 1,000 60 1.5 B 1,200 58 2 Determine the fair value of the share. A $60 B $58 C $58.5 D $56

The correct answer is (B). The principal market is the market with the greatest volume and maximum level of activity. In this question, Market B is the principal market and the share's price on this market would be the fair value. 1000 x (60-1.5) = 58,500 1200 x (58-2) = 67,200

When valuing certain financial instruments, a company that has elected the fair value measurement option must apply the accounting measurement based on which of the following criteria? A A portion of an asset or liability B Instrument-by-instrument basis C Type-by-type basis D At the entity level

The correct answer is (B). ASC 825-10-25-2 states the fair value measurement option is irrevocable, must be applied on an instrument-by-instrument basis, and must be applied to the entire instrument as a whole. (A) is incorrect because the entire asset or liability must be measured at fair value. (C) is incorrect because fair value must be measured on an instrument-by-instrument basis for each type that is to be valued under fair value measurement. (D) is incorrect because fair value must be measured for each instrument and not at the entity level.

At the commencement date of an operating lease, a lessor should A Subtract initial direct costs B Defer initial direct costs C Recognize initial direct costs D None of the above

The correct answer is (B). At the commencement date, a lessor should **defer initial direct costs**. Initial direct costs associated with the lease are deferred and amortized over the term of the lease as the lease income is recognized. After the commencement date, a lessor should recognize all of the following: The lease payments as income in profit or loss over the lease term on a straight-line basis. Variable lease payments as income in profit or loss in the period in which the changes in facts and circumstances on which the variable lease payments are based occur Initial direct costs as an expense during the lease period on the same basis as income from the lease.

During the year, Pitt Corp. incurred costs to develop and produce a routine, low-risk computer software product, as follows: Completion of detail program design$13,000Costs incurred for coding and testing to establish technological feasibility10,000Other coding costs after the establishment of technological feasibility24,000Other testing costs after the establishment of technological feasibility20,000Costs of producing product masters for training materials15,000Duplication of computer software and training materials from product masters (1,000 units)25,000Packaging product (500 units)9,000 In Pitt's December 31 balance sheet, what amount should be reported in inventory? A $25,000 B $34,000 C $40,000 D $49,000

The correct answer is (B). Computer software developed to sale, lease or market as a product: RD costs are expensed. These are costs incurred prior to technological feasibility (technological feasibility is established upon completion of a detailed program or design or completion of a working model). The completion of detail program design for $13,000 and costs incurred for coding and testing to establish technological feasibility of $10,000 are expensed. Costs associated with converting a technologically-feasible program into a final commercial form is capitalized. $24,000 in coding costs after the establishment of technological feasibility, $20,000 in testing costs after the establishment of technological materials, and costs of producing product masters for training materials of $15,000 are all capitalized. Costs incurred after software sales begin are inventoried - this will include duplication of computer software and training materials from product masters for $25,000 and packaging product for $9,000. The year-end balance sheet would report inventory of $34,000 (i.e. $25,000 + $9,000).

A U.S. company purchased inventory on account at a cost of 1,000 foreign currency units (FCU) from a non-U.S. company on November 15, to be paid on December 15. The FCU is valued at $0.85 on November 15 and at $0.90 on December 15. The journal entry to record payment on December 15 should include which of the following? A Debit inventory and credit cash for $850 B Debit exchange gains and losses and credit accounts payable for $50 C Debit accounts payable and credit exchange gains and losses for $50 D Debit accounts payable and credit cash for $850

The correct answer is (B). Purchased inventory would have been recorded for $850 on November 15th because of $0.85 FCU value x 1,000. On December 15th, with an exchange rate of $0.90, it will require $900 to make the payment. Accounts payable will be increased with a credit of $50 and an exchange loss will be recognized with a debit of $50. Exchange loss $50A/P $50 (A) is incorrect because inventory will be debited at the purchase date (C) is incorrect because there is a loss, the reverse of this entry must be passed. (D) is incorrect because the amount should include exchange gain or loss

If Tron Inc. a US-based entity has accounts payable valued in foreign currency, it should be adjusted forchanges in the: A Spot rate and the foreign exchange gains or losses are reported as operating gains or losses on the income statement. B Spot rate and the foreign exchange gains or losses are reported as non-operating gains or losses on the income statement. C Average rate and the foreign exchange gains or losses are reported as operating gains or losses on the income statement. D Average rate and the foreign exchange gains or losses are reported as non-operating gains or losses on the income statement.

The correct answer is (B). When an entity with the US Dollar as a functional currency engages in a transaction in a foreign currency, it remeasures the transaction using the spot rate as of the transaction date. **If the entity has any monetary assets or liabilities (e.g., cash, accounts receivable, accounts payable)** valued in the foreign currency, these need to be adjusted for changes in the spot rate and the foreign exchange gains or losses are reported as **non-operating gains or losses on the Income Statement.**

Mentor Co., a U.S. corporation, owned 100% of a Swiss corporation. The Swiss franc is the functional currency. The remeasurement of Mentor's financial statements resulted in a $25,000 gain at year-end. The translation of the financial statements resulted in a $40,000 gain at year-end. What amount should Mentor recognize as foreign currency gain in its income statement? A $0 B $25,000 C $40,000 D $65,000

The correct answer is (B). When the company executes a transaction in a currency other than the one in which the books are maintained, the transaction is remeasured to the functional currency. This remeasurement gain or loss is recognized in the income statement. If the company maintains its books in a foreign currency, but issues its financial statements using a different currency, all transactions are *translated* to the reporting currency. This translation gain or loss is *recognized in the other comprehensive income*. A *remeasurement* gain of $25,000 will be *recognized in the income statement*. A translation gain of $40,000 will only be recognized in the other comprehensive income.

Mentor Co., a U.S. corporation, owned 100% of a Swiss corporation. The Swiss franc is the functional currency. The remeasurement of Mentor's financial statements resulted in a $25,000 gain at year-end. The translation of the financial statements resulted in a $40,000 gain at year-end. What amount should Mentor recognize as foreign currency gain in its income statement? A $0 B $25,000 C $40,000 D $65,000

The correct answer is (B). When the company executes a transaction in a currency other than the one in which the books are maintained, the transaction is remeasured to the functional currency. This remeasurement gain or loss is recognized in the income statement. If the company maintains its books in a foreign currency, but issues its financial statements using a different currency, all transactions are translated to the reporting currency. This translation gain or loss is recognized in the other comprehensive income. A remeasurement gain of $25,000 will be recognized in the income statement. A translation gain of $40,000 will only be recognized in the other comprehensive income.

A company leases a machine from Leasing, Inc. on January 1, year 1. The lease terms include a $100,000 annual payment beginning January 1, year 1. The machine's fair value is $500,000 and the residual value is estimated at $20,000. The company guarantees the residual value. The useful life of the machine is six years, and the lease term is five years. The implicit rate of interest is 6% and is known by the company. The following present value factors are provided: Five years Six years Present value of $1 at 6% 0.7473 0.705 Present value of an annuity due at 6% 4.4651 5.2124 Present value of an ordinary annuity at 6% 4.2124 4.9173 What is the value of the machine in the company's balance sheet at lease inception? A $446,510 B $461,456 C $520,000 D $535,340

The correct answer is (B). With a lease term equal to 5 years of the asset's 6 years useful life, as the lease term is for the major part of the remaining economic life of the underlying asset, the lease is a finance lease and will result in the recognition of an asset and a liability equal to the present value of the minimum lease payments. The guaranteed residual value on the machine is a one-time payment at the end of the lease term calculated at the PV of $1 at 6% for 5 years. Annual payments for leasing the machine$100,000 PV of an annuity at 6% for 5 Years 4.4651 PV of annual payments $446,510 Residual value of machine $20,000 PV 0f $1 at 6% 0.7473 PV of residual value of the machine $14,946 Value of machine at lease inception (PV of annual payments + PV of residual value of the machine) $461,456

n a sale-leaseback transaction, the seller-lessee retains the right to substantially all of the remaining use of the equipment sold. The profit on the sale should be deferred and subsequently amortized by the lessee when the lease is classified as: #Finance Lease/Operating lease A No/Yes B No/No C Yes/Yes D Yes/No

The correct answer is (B). There is no gain deferral under either a finance lease or an operating lease. According to ASC 842, the transfer of the asset must meet the requirements for a sale per the Revenue Recognition standards. If there is no sale for the seller-lessee, the buyer-lessor also does not account for a purchase. Any consideration paid for the asset is accounted for as a financing transaction by both the seller-lessee and the buyer-lessor. #If the leaseback is a finance lease from seller-lessee's perspective, the transfer of the asset is not a sale and therefore, no gain would be recognized.# #However, if the leaseback is an operating lease from seller-lessee's perspective, transfer of the asset is a sale and gain would be recognized immediately.#

A company enters into a three-year operating lease agreement effective January 1, year 1. The amounts due at the end of each year are $25,000 in year 1, $30,000 in year 2, and $35,000 in year 3. The rate implicit in the lease is 10%. What amount, if any, is the related liability on the first day of year 2? Additional Information: Present Value of $ 1 at 10% for 1 period = 0.9091 Present Value of $ 1 at 10% for 2 period = 0.8264 Present Value of $ 1 at 10% for 3 period = 0.7513 A $0 B $73,816 C $56,198 D $31,818

The correct answer is (C) At commencement, the initial measurement of the lease liability (regardless of lease classification) is calculated as the present value of the lease payments not yet paid by using the lease term and discount rate determined at lease commencement. Rate Implicit in the Lease is 10%. So lease liability will be recorded at present value of all minimum lease payments $25,000 0.9091 $22,727 $30,000 0.8264 $24,793 $35,000 0.7513 $26,296 Total$73,816 The Journal Entry would be: Dr. ROU Asset $73,816 Cr. Lease Liability $73,816 After lease commencement, a lessee would recognize a single lease cost in the income statement on a straight-line basis. The total remaining lease cost on the commencement date would be $90,000 which is calculated as the total lease payments ($25,000 + $30,000 +$35,000). This remaining lease cost is recognized on a straight-line basis over the remainder of the lease term (i.e., Lessee would recognize $30,000 in each period, which is calculated as $90,000 ÷ 3). Lease Liability Year Beg. Bal Liab. Accretion Lease Pymt End. Bal. 1 73,816 7,382 (10%) (25,000) = 56,198 2 56,198 5,620 (30,000) = 31,818 3 31,818 3,182 (35,000) 0 ROU Asset Year Beg Bal Asset Red End. Bal. Lease Const 1. 73,816 (22,618=30000-7382) 51,198 30,000 2 51,198 (24,380) 26,818 30,000 3 26,818 (26,818) 0 30,000 The liability on the first day of Year 2 is $56,198.

Crossroads Co. chooses to report a financial asset at its fair value. The asset trades in two different markets; however, neither market is the principal market for the financial asset. In the first market, sales proceeds are $76, which is net of transaction costs of $6. In the secondary market, the sales proceeds are $80, which is net of transaction costs of $1. What amount should Crossroads report as the fair value of the asset? A $76 B $80 C $81 D $82

The correct answer is (C). According to the facts of the question, the asset trades in two different markets; however, neither market is the asset's principal market, which is the market with the greatest volume and level of activity. If the asset's principal market cannot be identified, the most advantageous market should be used when determining the fair value of a financial asset. The most advantageous market would be the one which generates the highest net price, after subtracting transaction costs. The secondary market in the question generates the highest net price of $80 after subtracting the transaction costs; therefore, this market should be used for fair value purposes. The fair value amount will exclude any transaction costs, which results in a fair value of $81 for the asset. Note: Transaction costs will not be incorporated into the fair value. (A) is incorrect because $76 is the net proceeds in the first market which is not the most advantageous market when compared to the secondary market. (B) is incorrect because $80 is the fair value after adjusting for transaction costs in the secondary market. (D) is incorrect because first market is not the most advantageous market after considering the transaction costs (i.e. $82 = $76 + $6).

ASU 2016-02 covers leases of all property, plant, and equipment, as well as A Assets under construction B Biological assets C Intangible assets D None of the above

The correct answer is (D) According to ASU 2016-02, a Lease is a contract that gives a customer the right to control the use of an identified asset (PP&E) for a period of time in exchange for some form of consideration. Control exists if the customer has: 1. The right to obtain substantially all of the economic benefits from the asset. 2. The right to direct (control) the use of the asset. It includes leases of all PP&E, but excludes leases for: 1.Intangible assets 2.Exploring for minerals, gas, oil, etc. 3.Biological assets 4.Inventory 5.Assets under construction

On 12/31, a share trades at $100 on market A and at $102 on market B. If the transaction costs are $2 in A and $3 in B, determine the fair value of the share if none of the markets qualify to be the principal market. A $100 B $99 C $98 D $102

The correct answer is (D). Fair value is a market-based (not an entity-based) measurement. The objective is to estimate the price that would be received to sell an asset or paid to transfer a liability in an orderly transaction between market participants at the measurement date (that is, an exit price from the perspective of a market participant that holds the asset or owes the liability). The term "orderly transaction" assumes that the item has been exposed to the marketplace prior to the measurement date for a reasonable period of time (not a forced transaction). The principal market, or otherwise most advantageous market, is determined from the standpoint of the entity that holds the asset or liability and requires the entity to consider the market in which it conducts its highest volume or level of activity. The net realizable value is considered when using the most advantageous market approach. The NRV is $98 and $99 in markets A and B, respectively. Because NRV is the maximum in market B, the share price in that market is the fair value of the security.

A company owns land and a building that houses its manufacturing operations. When the company purchased the manufacturing facility 10 years ago, the purchase price allocated to the land account was $120,000. The manufacturing facility is located in an area that was once the site of many factories. The owners of many of the neighboring factories have recently sold their facilities to residential real estate developers. The company's land is also suitable for residential development. The estimated current value of the land as part of the manufacturing facility is $150,000. The estimated current value of the land as an undeveloped investment is $130,000, and the current value of the land as part of a residential development would be $180,000. What is the fair value of the land? A $120,000 B $130,000 C $150,000 D $180,000

The correct answer is (D). The fair value of an asset is determined in the most **advantageous market**, in which the amount realized for the asset is the maximum or in which the costs to sell are the least. In the given problem, there are three advantageous markets for the land: As a manufacturing facility As an undeveloped investment As part of a residential development However, the amount realized for the land as part of a residential development is the maximum value vs all other markets. The fair value of the said land is $180,000.


संबंधित स्टडी सेट्स

CA Real Restate Vocabulary - Chapter 2 Laws of agency and fiduciary duties

View Set

CIST1601-New Chapter 8 Review Questions

View Set

Physics: Rotational, Projectile, Satellite Motion, Gravity

View Set

Rock and Roll Midterm Studyguide

View Set

OB Chapt 10 Fetal Development & Genetics

View Set